Download as pdf or txt
Download as pdf or txt
You are on page 1of 50

TEST -12 (3189)

VISION IAS
PT TEST-12
(3189)
Ancient Indian History, Art and
Culture and Medieval India

2021
https://t.me/UPSC_PDF Download From > https://upscpdf.com https://t.me/UPSC_PDF
Test 12 2021

1. Consider the following statements: 4. With reference to the recently launched


1. This style was developed under the Naval Innovation and Indigenisation
patronage of Adil Shah. Organisation (NIIO), consider the following
2. Its architecture is unique in the use of statements:
three-arched facade and bulbous dome. 1. It is a three-tiered organization aimed at
3. Gol Gumbad is one the finest example of boosting innovation in defence through
this architecture. collaboration with academia.
Which of the following schools of 2. Under NIIO, a Naval Technology
architecture is best described by the above Acceleration Council (N-TAC) will be
statements? set up as an apex decision making body.
(a) Malwa school Which of the statements given above is/are
(b) Jaunpur School correct?
(c) Bijapur School (a) 1 only
(d) Bengal School (b) 2 only
(c) Both 1 and 2
2. Which of the following folk dances are form (d) Neither 1 nor 2
of mask dance?
1. Singhi chham
5. Consider the following pairs:
2. Mayurbhanj Chhau
Medieval Port Region
3. Bihu
1. Tamralipti Maharastra
4. Butta Bommalu
2. Chaul Bengal
5. Padayani
3. Masulipatnam Andhra Pradesh
Select the correct answer using the code
Which of the pairs given above is/are
below.
correctly matched?
(a) 1 and 2 only
(a) 1 and 2 only
(b) 2 and 5 only
(b) 2 and 3 only
(c) 1, 4 and 5 only
(c) 3 only
(d) 1, 3, 4 and 5 only
(d) 1, 2 and 3

3. Consider the following statements regarding


6. With reference to Magadha success, consider
the Satavahanas rulers of Ancient India:
the following statements:
1. Their social structure was completely
1. Presence of rich iron deposits.
patrilineal.
2. Efficient communication system.
2. Their coinage system consisted of gold
coins. 3. Impregnability of Rajgir.

3. Their official language was Prakrit. 4. Use of elephants in wars.

Which of the statements given above is/are Which of the statements given above is/are

correct? correct?

(a) 1 and 2 only (a) 1 and 3 only


(b) 3 only (b) 2, 3 and 4 only
(c) 2 and 3 only (c) 1 and 4 only
(d) 1 and 3 only (d) 1, 2, 3 and 4
2 ©Vision IAS

Google it:- https://upscpdf.com


https://t.me/UPSC_PDF Download From > https://upscpdf.com https://t.me/UPSC_PDF

7. With reference to Karkhanas in medieval 10. Recently NASA has given the status of an
India, consider the following statements: “ocean world” to the dwarf planet Ceres. In
1. Goods manufactured in the Karkhanas this context, which of the following is not a
were primarily meant for exports. criterion of classifying a celestial body as a
2. Apart from royal capitals, karkhanas dwarf planet?
were also established in provincial
(a) It must orbit around the Sun.
capitals.
(b) It must not be a satellite of another
3. During the Mughal era, Karkhanas were
celestial body.
headed by officers called Mir Saman.
(c) It must clear the neighborhood around
Which of the statements given above are
its orbit.
correct?
(a) 1 and 2 only (d) It must have enough mass for its gravity

(b) 2 and 3 only to pull it into a roughly spherical shape.

(c) 1 and 3 only


(d) 1, 2 and 3 11. Consider the following statements
about Nayakas in the Vijayanagara Empire:
8. Kalpa Sutra, a Jain ancient textbook 1. Nayakas enjoyed both military and
containing the biographies of the last two revenue powers.
Jain Tirthankaras is written by: 2. The rise of Nayakas led to
(a) Kautilya
the considerable rise of village
(b) Bhadrabahu
autonomy and administration.
(c) Ashvaghosha
3. The Nayakaship was hereditary.
(d) Sthulabhadra
Which of the statements given above are
correct?
9. With reference to the Dragonfly festival,
(a) 1, 2 and 3
recently seen in the news, consider the
following statements: (b) 1 and 3 only

1. It aims to educate and inform the public (c) 2 and 3 only

about the integral role of dragonflies and (d) 1 and 2 only


other insects.
2. It was organized by the World Wide 12. With reference to ancient India, the term
Fund along with the Bombay Natural Brahmadeya refers to
History Society. (a) a kind of tax levied on brahmanas
Which of the statements given above is/are (b) a land belonging to non brahman
correct?
peasants
(a) 1 only
(c) a land allotted to build temples dedicated
(b) 2 only
to Brahma.
(c) Both 1 and 2
(d) a tax free land gifted to brahmanas
(d) Neither 1 nor 2

3 www.visionias.in ©Vision IAS

Google it:- https://upscpdf.com


https://t.me/UPSC_PDF Download From > https://upscpdf.com https://t.me/UPSC_PDF

13. What could have been the possible reasons 16. One of the most remarkable exploits of
for the Delhi Sultans to covet the Gujarat Rajendra I was his march across Kalinga to
and Malwa region? Bengal where his armies defeated two local
1. Fertility of the land kings. To commemorate this occasion, he
2. Accumulated wealth of local rulers built a new capital Gangaikondacholapuram
3. Supply of horses on the mouth of which river?
Select the correct answer using the code (a) Godavari
given below. (b) Ganga
(a) 1 and 2 only (c) Kaveri
(b) 2 and 3 only (d) Krishna
(c) 1 and 3 only
(d) 1, 2 and 3 17. Which of the following philosophies were
materialistic in their outlook?
14. Consider the following pairs 1. Samkhya
Folk Music State 2. Mimamsa
1. Maand Gujarat 3. Vaisheshika
2. Khongjom Parba Manipur 4. Lokayata
3. Wanawan Jammu & Kashmir Select the correct answer using the code
4. Baul Rajasthan given below.
Which of the pairs given above are correctly (a) 1 and 3 only
matched? (b) 1, 3 and 4 only
(a) 1 and 4 only (c) 2, 3 and 4 only
(b) 2 and 3 only (d) 1, 2, 3 and 4
(c) 2, 3 and 4 only
(d) 1, 2 and 3 only 18. Consider the following statements with
regard to Mauryan art and culture?
15. This Buddhist council was held under the 1. Marble was used on an extensive scale.
patronage of king Kanishka and presided by 2. The pillars were monolithic and highly
Vasumitra. At this council, Buddhism polished.
underwent a split into two sects - Mahayana 3. They introduced stone masonry on an
and Hinayana. Which Buddhist Council is intensive scale.
being described in the paragraph given Which of the statements given above is/are
above? correct?
(a) First (a) 1 and 2 only
(b) Second (b) 2 and 3 only
(c) Third (c) 2 only
(d) Fourth (d) 1 and 3 only

4 www.visionias.in ©Vision IAS

Google it:- https://upscpdf.com


https://t.me/UPSC_PDF Download From > https://upscpdf.com https://t.me/UPSC_PDF

19. The Karmaprabhrita and the 23. Consider the following statements with
Kashayaprabhrita are the canonical works reference to Buddhism and Jainism:
recognized by: 1. Jain orders admitted only men whereas
(a) Mahayana Buddhism Buddhist sanghas admitted both men and
(b) Hinayana Buddhism women.
(c) Shvetambara Jains 2. Both believed in following a middle path
(d) Digambara Jains to attain enlightenment.
Which of the statements given above is/are
20. 'Agriota', a new technology-driven agri- correct?
commodity trading and sourcing e-market (a) 1 only
platform, aims to connect Indian farmers (b) 2 only
with the food industry of which of the (c) Both 1 and 2
following countries? (d) Neither 1 nor 2
(a) United Arab Emirates
(b) United Kingdom 24. Which of the following states has recently

(c) United States of America passed a resolution to bring the state under
Sixth Schedule?
(d) Japan
(a) Assam
(b) Chhattisgarh
21. With reference to the reign of Firoz Shah
(c) Nagaland
Tughlaq, consider the following statements:
(d) Arunachal Pradesh
1. Jizya became a separate tax.
2. Translation of Hindu scriptures to
25. Consider the following statements with
Persian was banned.
reference to 'Virashaiva Tradition':
3. Inhuman punishments for petty offenses
1. It began during the age of the Chalukya
became common.
dynasty.
Which of the statements given above is/are
2. Its followers did not believe in the cycle
correct?
of rebirth.
(a) 1 only
3. Sayings of men and women saints of
(b) 2 and 3 only
Virashaiva tradition are known as
(c) 1 and 3 only
Abhangas.
(d) 1, 2 and 3 Which of the statements given above is/are
correct?
22. What factors facilitated the Turkish invasion (a) 1 and 2 only
of Delhi in the 11th -12th century? (b) 2 only
1. Better organization skill of the Turkish (c) 1 and 3 only
army (d) 1, 2 and 3
2. Artillery Supremacy of invaders
3. Internal conflicts of Indian rulers 26. Sallekhana or santhara a practice seen is
Select the correct answer using the code related to.
given below. (a) Giving up of one’s household duties.
(a) 1 and 2 only (b) Ritual of fasting till death in jainism.
(b) 2 and 3 only (c) Receiting mantras with the turn of a
(c) 1 and 3 only wheel.
(d) 1, 2 and 3 (d) Veneration of Buddha and deities.
5 www.visionias.in ©Vision IAS

Google it:- https://upscpdf.com


https://t.me/UPSC_PDF Download From > https://upscpdf.com https://t.me/UPSC_PDF

27. 'South Atlantic Anomaly', a term is 31. Which of the following is/are correct
sometimes seen in news in the context of: regarding the social conditions during the
(a) magnetic field of the earth. Sultanate period?
(b) forest fires and climate change.
1. Women did not enjoy any property
(c) wind direction and weather patterns.
rights.
(d) salinity differences between oceans.
2. Forced religious conversion to Islam was

28. Consider the following statements in the widespread.


context of Purandaradasa: 3. There was a complete ban on the
1. He is called as "Sangeeta Pitamaha" of construction of temples.
Carnatic music. Select the correct answer using the code
2. His kirtans are called Vachanas. given below.
Which of the statements given above is/are
(a) 1 only
correct?
(b) 2 and 3 only
(a) 1 only
(b) 2 only (c) 1 and 3 only

(c) Both 1 and 2 (d) None


(d) Neither 1 nor 2
32. Arrange the following Chalcolithic culture
29. With reference to the economic and human from north to south.
life of the Indus Valley people, consider the 1. Ahar
following statements:
2. Kayath
1. They had dedicated bead-jewelry
3. Jorwe
making industries at Mohenjo-Daro and
Lothal. Select the correct answer using the code
2. The ornaments were used as materials of given below.
fashion and never buried with the dead. (a) 1-2-3
Which of the statements given above is/are (b) 2-1-3
correct? (c) 2-3-1
(a) 1 only
(d) 3-2-1
(b) 2 only
(c) Both 1 and 2
33. Consider the following pairs:
(d) Neither 1 nor 2
Island in News Country
30. Of the second Sangam, it is the only 1. Kyushu Japan
surviving work. It is a work on Tamil 2. Strait Sri Lanka
grammar but it also provides information on 3. Crete Greece
the political and socio-economic conditions Which of the pairs given above is/are
of the Sangam period. Which work is being
correct?
described in the above-given paragraph?
(a) 2 only
(a) Ettutogai
(b) 1 and 3 only
(b) Tolkappiyam
(c) Pathinenkilkanakku (c) 2 and 3 only
(d) Silappathikaram (d) 1, 2 and 3

6 www.visionias.in ©Vision IAS

Google it:- https://upscpdf.com


https://t.me/UPSC_PDF Download From > https://upscpdf.com https://t.me/UPSC_PDF

34. Consider the following statements about 36. Consider the following statements regarding
the art and architectural developments
Portuguese trade in India during the
during the Delhi Sultanate period:
Vijayanagara Empire: 1. Construction of Qutub Minar was started
1. Indian exports by the traders mainly by Qutbuddin Aibak.
2. New style of music known as Qawwalis
included saltpeter, sugar, rice, spices,
was introduced during the reign of Firoz
and textiles. Shah Tughlaq
2. Traders helped in transmitting potato, 3. Amir Khusrau’s famous work Khazain-
ul-Futuh deals with the rise of
tobacco, maize, cocoa, and peanuts from Ghiasuddin Tughlaq.
Central America to India. Which of the statements given above is/are
correct?
3. Trade in pepper, arms, and ammunition
(a) 1 only
and war horses in India was a royal (b) 1 and 2 only
monopoly and no private traders were (c) 2 and 3 only
(d) 1, 2 and 3
allowed to engage in the trade of these

goods. 37. Consider the following statements in the


context of Kushana coins:
Which of the statements given above is/are
1. Kanishka was the first to introduce gold
correct? coins in India.
(a) 3 only 2. Kushana's coins bore the images of both
Buddha and Shiva.
(b) 2 and 3 only
3. The purity of gold coins of kushana's
(c) 1 and 2 only period is lesser than that of Guptas.
(d) 1, 2 and 3 Which of the statements given above is/are
correct?
(a) 1 only
35. With reference to the Bhakti saint Lal Ded, (b) 2 only
(c) 1 and 2 only
consider the following statements:
(d) 3 only
1. She belonged to the Vaishnavite school

of bhakti. 38. Which of the following were practiced by


the people during the Vedic Period?
2. Her verses are known as Vakhs, which
1. They domesticated the horses.
are in the Kashmiri language, 2. They used of war chariots and spoked-
wheels.
Which of the statements given above is/are
3. They performed animal sacrifices.
correct? Select the correct answer using the code
(a) 1 only given below.
(a) 1, 2 and 3
(b) 2 only
(b) 2 and 3 only
(c) Both 1 and 2 (c) 1 and 2 only
(d) Neither 1 nor 2 (d) 3 only
7 www.visionias.in ©Vision IAS

Google it:- https://upscpdf.com


https://t.me/UPSC_PDF Download From > https://upscpdf.com https://t.me/UPSC_PDF

39. Which of the following were the market 42. Which of the following statements about the
reforms introduced by Alaudin Khalji? Sattriya dance form is/are correct?
1. It is one of the eight classical dance
1. Fixation of prices of commodities
forms in India.
2. Establishment of royal granaries
2. It was introduced by the Saint
3. Introduction of token currency Sankaradeva in the 15th century AD in
4. Establishment of standard weights and Assam.
measures 3. The focus of Sattriya recitals is to
narrate mythological stories of Shiva.
Select the correct answer using the code
4. It is performed generally by females.
given below. Which of the statements given above is/are
(a) 1, 2 and 4 only correct?
(b) 2 and 3 only (a) 3 only
(b) 3 and 4 only
(c) 1 and 4 only
(c) 1 and 2 only
(d) 1, 2, 3 and 4
(d) 1, 2, 3 and 4

40. Consider the following statements with 43. This dynasty dominated eastern India during
reference to the paintings of Ellora caves: the 8th and 9th centuries AD. They were the
great patrons of Buddhism and was
1. These paintings were painted during the
instrumental in reviving the Nalanda
same period when Ajanta paintings were
university. It was one of the kingdoms
drawn. engaged in a tripartite struggle over the city
2. These paintings are related to all three of Kannauj.
religions (Hinduism, Buddhism, and Which of the following dynasty is being
described in the above passage?
Jainism).
(a) Sena dynasty
Which of the statements given above is/ are (b) Pala dynasty
correct? (c) Rashtrakuta dynasty
(a) 1 only (d) Eastern Ganga dynasty

(b) 2 only
44. These paintings bear a resemblance to those
(c) Both 1 and 2
created in the Rock Shelters of Bhimbetka.
(d) Neither 1 nor 2 The unique feature is the use of very basic
graphic vocabulary like a circle, a triangle
41. With reference to the ancient history of and a square. All the paintings are done on a
red ochre or dark background, while the
India, the Prayag Prashasti inscription
shapes are white in colour. They are mainly
describes the:
done by tribal women in western India.
(a) territorial extent of the Satavahana Which of the following painting described in
empire. the paragraph given above?
(b) achievements of Samudragupta. (a) Kalamkari
(b) Pattachitra
(c) doctrines of Ajivika sect
(c) Warli
(d) Ashoka's victory over Kalinga (d) Gond
8 www.visionias.in ©Vision IAS

Google it:- https://upscpdf.com


https://t.me/UPSC_PDF Download From > https://upscpdf.com https://t.me/UPSC_PDF

Google it:- https://upscpdf.com


https://t.me/UPSC_PDF Download From > https://upscpdf.com https://t.me/UPSC_PDF

Google it:- https://upscpdf.com


https://t.me/UPSC_PDF Download From > https://upscpdf.com https://t.me/UPSC_PDF

57. With reference to the National Commission 60. Recently, an International Conference on
on Population consider the following ‘Aerosol Air Quality, Climate Change and
statements: Impact on Water Resources and Livelihoods
1. It is an ad-hoc body that is constituted in the Greater Himalayas’ was organized by:
(a) Aryabhatta Research Institute of
every year to estimate the regional
Observational Sciences
trends in population growth in India.
(b) Intergovernmental Panel on Climate
2. It is constituted under the Ministry of
Change
Health and Family Welfare.
(c) United Nations Environment Programme
Which of the statements given above is/are
(d) None of the above
correct?
(a) 1 only 61. The terms Samabhanga, Aabhanga,
(b) 2 only Atibhanga are related to
(c) Both 1 and 2 (a) temple architecture
(d) Neither 1 nor 2 (b) dance postures
(c) color combinations
58. Consider the following statements about the (d) musical ragas in Hindustani Music

Gandhara school of arts:


62. Consider the following pairs
1. This school for the first time depicted
Philosophy Founder
Buddha as a human form.
1. Dvaitadvaita Nimbaraka
2. Grey sandstone was used in it.
2. Suddhadvaita Madhavacharya
3. Calm and expressiveness remains the
3. Vishistadvaita Ramanujacharya
central point of these images. 4. Dvaita Vallabhacharya
Which of the statements given above is/are Which of the pairs given above are correctly
correct? matched?
(a) 1 and 2 only (a) 2 and 4 only
(b) 2 and 3 only (b) 1 and 4 only
(c) 3 only (c) 2 and 3 only
(d) 1, 2 and 3 (d) 1 and 3 only

63. With reference to Thirukkural, consider the


59. This school of philosophy believes that
following statements:
nothing is acceptable unless it is in
1. It was composed as a commentary to the
accordance with reason and experience. It is
Sangam literature.
considered as a technique of logical
2. It is divided into three parts related to
thinking. Which of the following school of virtue, government and love.
philosophy is being described in the Which of the statements given above is/are
paragraph given above? correct?
(a) Samkhya (a) 1 only
(b) Nyaya (b) 2 only
(c) Vaisheshika (c) Both 1 and 2
(d) Vedanta (d) Neither 1 nor 2

11 www.visionias.in ©Vision IAS

Google it:- https://upscpdf.com


https://t.me/UPSC_PDF Download From > https://upscpdf.com https://t.me/UPSC_PDF

64. In the context of the Mauryan period, 67. In the context of various travellers who
Dharmasthiya and Kantakasodhana were visited India during the reign of different
(a) Priests for the interpretations of laws. dynasties, consider the following pairs:
(b) Law courts. Travelers Dynasty
(c) Officers in charge of propagation of 1. Ibn-Batuta Tughlaqs
Ashoka’ s Dhamma. 2. Marco Polo Vijayanagar
(d) Minister in charge for the
3. Niccolò de' Conti Kakatiyas
superintendence of religious and social
Which of the pairs given above is/are
affairs.
correctly matched?
(a) 2 only
65. Consider the following statements with
(b) 2 and 3 only
regard to Chandragupta II:
(c) 1 only
1. He founded the Nalanda university.
(d) 1, 2 and 3
2. He was the last ruler of the Gupta
dynasty.
3. Harisena was his court poet. 68. It is a traditional folk theatre form of Kerala

Which of the statements given above is/are is celebrated in the month of Vrischikam

correct? (November-December). It is usually


(a) 1 only performed only in the Kali temples of
(b) 1 and 3 only Kerala, as an oblation to the Goddess. It
(c) 2 and 3 only depicts the triumph of goddess Bhadrakali
(d) None over the asura Darika.
Which of the following traditional Indian
66. With reference to the cultural history of theatre forms given below is best described
Mughals in India, consider the following by the above passage ?
statements: (a) Mudiyettu
1. The Indo-Islamic architecture in India
(b) Bhaona
began with the rule of Mughals.
(c) Yakshagaana
2. The decorative motifs in Akbar's tomb in
(d) Therukoothu
Sikandra include elements like lotus and
swastika.
69. Tuti-Nama, Hamza- Nama, Anvar- i- suhaili
3. Buland Darwaza was built by Akbar to
and Gulistan of sadi are the famous
commemorate his victory over Gujarat.
manuscripts associated with which of the
Which of the statements given above is/are
following Mughal ruler?
correct?
(a) 1 only (a) Jahangir

(b) 3 only (b) Babur

(c) 2 and 3 only (c) Akbar


(d) 1 and 3 only (d) Shahjahan

12 www.visionias.in ©Vision IAS

Google it:- https://upscpdf.com


https://t.me/UPSC_PDF Download From > https://upscpdf.com https://t.me/UPSC_PDF

70. With reference to Jal Jeevan Mission, 73. The paintings of flying apsaras and scene of
consider the following statements: Mahajanaka Jataka are depicted in which of
1. It aims at providing a Functional the following ancient sites?
Household Tap Connection (FHTC) to (a) Bagh caves
every rural household of the country by (b) Ellora caves
2024. (c) Elephanta caves
2. Under the mission, a Paani Samiti will (d) Ajanta caves
be established at the district level
74. Consider the following statements:
to function as a public utility for
1. Tempera paintings are done on dry
monitoring the implementation.
plaster, while frescos are done on wet
3. The mission also promotes the
plaster.
digitisation of water supply 2. Both styles are found in Ajanta and
infrastructure. Bagh paintings.
Which of the statements given above is/are Which of the statements given above is/are
correct? correct?
(a) 1 only (a) 1 only
(b) 2 and 3 only (b) 2 only
(c) 1 and 3 only (c) Both 1 and 2
(d) 1, 2 and 3 (d) Neither 1 nor 2

71. Which of the following functions were 75. Consider the following statements:
performed by the village assembly, i.e. 1. Kavirajamarga is a famous Jaina text
Sabha during the later Cholas? written by Ravikriti.
1. Raise loans for the village 2. Chandragupta Maurya was converted to
Jainism by Bhadrabahu.
2. Levy taxes
3. Jainism was patronized by Rashtrakuta
3. Maintenance of charitable institutions
and Kadamb dynasties.
4. Exercise ownership rights over land
Which of the statements given above is/are
Select the correct answer using the code
correct?
given below. (a) 1 only
(a) 1, 2 and 3 only (b) 2 and 3 only
(b) 2 and 3 only (c) 1 and 2 only
(c) 1 and 4 only (d) 1, 2 and 3
(d) 1, 2, 3 and 4
76. With reference to Yoga philosophy, consider
72. Consider the following pairs: the following statements:
Mahajanapadas Capital 1. It does not believe in the existence of
1. Anga : Champa God.
2. Vajji : Vaishali 2. It describes the way to get rid of all the
3. Avanti : Kaushambi kleshas or sufferings in human life.
4. Magadha Rajgriha 3. It believes in the union of mind, body
Which of the pairs given above is/are and soul.
Which of the statements given above is/are
correct?
correct?
(a) 1 and 4 only
(a) 1 and 3 only
(b) 2 and 3 only
(b) 2 and 3 only
(c) 1, 2 and 4 only
(c) 2 only
(d) 1 and 2 only (d) 1, 2 and 3
13 www.visionias.in ©Vision IAS

Google it:- https://upscpdf.com


https://t.me/UPSC_PDF Download From > https://upscpdf.com https://t.me/UPSC_PDF

77. Consider the following statements with 80. Which of the following are the chief
reference to Indian dance elements: characteristics of human life in the Neolithic
1. Nritya refers to the basic dance steps, Age?
performed rhythmically but devoid of 1. Crop cultivation
any expression or mood. 2. Animal husbandry
2. Natya means dramatic representations. 3. Fixed Abode
3. Nritta refers to the sentiment and the 4. Tools of polished stone
emotions evoked through dance. 5. Extensive knowledge of Metallurgy
Which of the statements given above is/are Select the correct answer using the code
correct? given below.
(a) 1 and 3 only (a) 1 and 3 only
(b) 1, 3, 4 and 5 only
(b) 1 only
(c) 2, 4 and 5 only
(c) 2 only
(d) 1, 2, 3 and 4 only
(d) 1, 2 and 3

81. Consider the following statements regarding


78. With reference to the recently launched
the sites of Indus Valley Civilisation (IVC):
Nationally Determined Contributions
1. Chanhudaro was a city without citadel.
(NDC)–Transport Initiative for Asia (TIA),
2. Dholavira city was divided into three
consider the following statements:
parts.
1. It aims to promote a comprehensive
3. Rakhigarhi is the largest Indian site of
approach to decarbonize transport in IVC.
India, Vietnam, and China. Which of the statements given above is/are
2. In India, the implementing agency for correct?
the programme on behalf of the (a) 1 and 2 only
government is NITI Aayog. (b) 3 only
Which of the statements given above is/are (c) 2 and 3 only
correct? (d) 1, 2 and 3
(a) 1 only
(b) 2 only 82. Which of the following socio-economic
(c) Both 1 and 2 conditions were associated with the Rigvedic
(d) Neither 1 nor 2 period?
1. There existed women slaves during the
79. Consider the following statements regarding Vedic period.
Buddhism: 2. There was absence of a serving order in
1. It believed in the transmigration of souls. form of Sudras.
2. It advocated the path of extreme 3. A war booty was the substantial source
austerity. of revenue to the chief.
3. It preached that the ultimate aim in life 4. The most common gift given to the
is to attain nirvana. priests was cereals.
Which of the statements given above is/are Select the correct answer using the code
correct? given below.
(a) 1 and 4 only
(a) 1 and 3 only
(b) 1, 2 and 3 only
(b) 3 only
(c) 2, 3 and 4 only
(c) 1 and 2 only
(d) 1 and 3 only
(d) 1, 2 and 3
14 www.visionias.in ©Vision IAS

Google it:- https://upscpdf.com


https://t.me/UPSC_PDF Download From > https://upscpdf.com https://t.me/UPSC_PDF

83. Consider the following sites: 86. Consider the following statements with
1. Daimabad reference to the Kishangarh school of
2. Kotdiji
painting:
3. Suktagendor
1. This style of painting essentially depicts
4. Surkotada
Which of the Harappan sites given above lie Radha and Krishna as divine lovers.
in India? 2. Nihal Chand was the famous painter
(a) 1 and 4 only under this school.
(b) 1, 2 and 4 only
3. Bani Thani is a famous painting
(c) 1 and 2 only
(d) 2 and 3 only associated with this school.
Which of the statements given above are
84. Consider the following statements with correct?
reference to Hindustani classical music:
(a) 1 and 2 only
1. Dhrupad as a classical form of music
(b) 2 and 3 only
reached its zenith in the court of emperor
Akbar. (c) 1 and 3 only
2. The origin of Khayal style of music is (d) 1, 2 and 3
attributed to Tansen.
3. Comparable to the vanis of the dhrupads,
87. Consider the following pairs:
gharanas are part of the khyal music.
Which of the statements given above is/are Book Author

correct? 1. Mahabhasya Patanjali


(a) 2 only 2. Mudrakshasa Vishakhadutta
(b) 1 and 2 only
3. Brihatkatha Gunadhya
(c) 1 and 3 only
Which of the pairs given above are correctly
(d) 1, 2 and 3
matched?
85. Consider the following statements with (a) 1 and 3 only
reference to the religious policy of Akbar: (b) 2 and 3 only
1. He abolished the pilgrim tax on bathing
(c) 1 and 2 only
at holy places.
2. He abolished the practice of forcibly (d) 1, 2 and 3

converting prisoners of war to Islam


3. He founded a new religion known as 88. Price Monitoring and Resource Units
‘Din-i-Ilahi’ based on the common
(PMRU) are societies set up at the State/UT
points of all religions.
level to monitor the prices of:
Which of the statements given above are
correct? (a) Drugs and Pharmaceuticals.
(a) 1 and 2 only (b) Fruits and Vegetables.
(b) 2 and 3 only (c) Food crops and Cereals.
(c) 1 and 3 only
(d) Kerosene, Petrol and Diesel
(d) 1, 2 and 3
15 www.visionias.in ©Vision IAS

Google it:- https://upscpdf.com


https://t.me/UPSC_PDF Download From > https://upscpdf.com https://t.me/UPSC_PDF

89. Originating from Himachal Pradesh, this 92. Which of the following Sufi orders followed
martial art is a mixture of sports and culture. the Islamic Law (Sharia)?
It takes place during the Baisakhi festival in 1. Chishti
April every year. The martial art relies on a 2. Qadiri
player’s skill of archery. This martial art can 3. Qalandars
also be dated back to Mahabharata. 4. Naqshbandi

Which of the following martial art is Select the correct answer using the code

described in the above passage? given below.

(a) Gatka (a) 1, 2 and 4 only

(b) Thoda (b) 2 and 3 only

(c) Mardani Khel (c) 1 and 4 only

(d) Sarit Sarai (d) 1, 2, 3 and 4

93. Consider the following statements in the


90. In the context of puppetry, which of the
context of the literature of the Gupta period :
statements given below is correct?
1. Prakrit was the court language during
(a) Tholu Bommalata is the traditional
Guptas.
shadow puppetry of Andhra Pradesh.
2. The plays produced in the Gupta period
(b) Kathputli is a variant of shadow
were mostly comedies.
puppetry in Rajasthan.
3. The celebrated Sanskrit drama
(c) Pavakoothu is a traditional glove
Mricchakatika was composed during this
puppetry of Tamilnadu.
time.
(d) Kundhei is a form of string puppetry
Which of the statements given above is /are
famous in Karnataka.
correct?
(a) 2 and 3 only
91. With reference to Ashoka's Dhamma,
(b) 3 only
consider the following statements:
(c) 1 and 3 only
1. It was regarded as sectarian faith.
(d) 2 only
2. Its broad objective was to preserve the
social order.
94. Which of the following statements regarding
3. It was intended to attain nirvana which
the Gupta period is not correct?
was the goal of Buddhist teaching. (a) The position of women became
Which of the statements given above is/ are miserable.
correct? (b) Caste system became rigid.
(a) 1 and 3 only (c) Scientific fervor underwent a drastic
(b) 2 and 3 only reduction under the Guptas.
(c) 2 only (d) There was less or no state interference in
(d) 1, 2 and 3 the individual’s life.

16 www.visionias.in ©Vision IAS

Google it:- https://upscpdf.com


https://t.me/UPSC_PDF Download From > https://upscpdf.com https://t.me/UPSC_PDF

95. Consider the following statements regarding 98. K V Kamath committee, recently seen in
Kalinga school of temple architecture: news, is related to:
1. Large gopurams and water tanks are (a) restructuring of stressed loans by banks.
present in temple premises.
(b) role of APMCs in agricultural
2. It is characterised by 'mandaps' or dance
procurement and marketing.
pavilions.
3. It is a sub-style of Dravida style of (c) assessment of fair price remuneration for
temple architecture. sugarcane crops
Which of the statements given above is/are (d) enhancement of loan disbursement to
correct? MSMEs in India.
(a) 2 only
(b) 2 and 3 only
99. Project P-75I, recently seen in news, is
(c) 1 and 3 only
(d) 1, 2 and 3 related to:
(a) procurement of submarines by Indian
96. Consider the following statements about Navy.
Mughal paintings: (b) commemoration of 75 years of Indian
1. They were prominently religious in independence.
character.
(c) increasing irrigation to 75% arable land
2. These paintings reached pinnacle under
in India.
the patronage of Aurangzeb.
3. Use of brilliant colours and (d) achieving high female rural literacy rate
ornamentation were the characteristic in India.
features.
Which of the statements given above is/are 100. With reference to India's trade in defence
correct?
and arms consider the following statements:
(a) 1 and 3 only
1. During the last five years, India has been
(b) 1 and 2 only
(c) 3 only the largest importer of arms in the world.
(d) 1, 2 and 3 2. The largest share of Indian arms import
is received from the United States of
97. Which of the following is/are the America.
architectural forms of Nagara temples? Which of the statements given above is/are
1. Latina
correct?
2. Phamsana
(a) 1 only
3. Valabhi
Select the correct answer using the code (b) 2 only
given below. (c) Both 1 and 2
(a) 1 only (d) Neither 1 nor 2
(b) 1 and 3 only
(c) 2 and 3 only
(d) 1, 2 and 3

Google it:- https://upscpdf.com


https://t.me/UPSC_PDF Download From > https://upscpdf.com https://t.me/UPSC_PDF

VISIONIAS
www.visionias.in
ANSWERS & EXPLANATIONS
GENERAL STUDIES (P) TEST – 3189 (2021)

Q 1.C
• Bijapur School
o Under the patronage of Adil Shah, Bijapur style or the Deccan style of architecture developed. He
constructed a number of mosques, tombs and palaces which were unique in the use of 3-arched
facade and bulbous dome, and were almost spherical with a narrow neck. He also introduced the use
of cornices. A special feature of the Bijapur school was the treatment of its ceilings, which were
without any apparent support. Iron clamps and a strong plaster of mortar were used to give
strength to the buildings. The walls were decorated by rich carvings.
o Example: Gol Gumbad (the mausoleum of Adil Shah)
• Malwa School
o The cities of Dhar and Mandu in the Malwa plateau became prominent seats of architecture. The most
prominent feature of the buildings here was the use of different coloured stones and marbles. result
of European influence and were decorated by stylised use of arches and pillars.
o Malwa School of Architecture, also known as the Pathan School of Architecture is also one of
the finest specimens of environmental adaptation.
o Examples: Rani Roopmati Pavilion, Jahaz Mahal, Ashrafi Mahal, etc.
• Jaunpur School
o Patronised by the Sharqi rulers, Jaunpur became a centre of great art and cultural activity. This style
of architecture also came to be known as Sharqi style and also avoided the use of minars, as the
Pathan style. A unique feature of the buildings here is the use of bold and forceful characters painted
on huge screens in the centre and side bays of the prayer hall.
o Example: Atala Mosque, Jaunpur.
• Bengal School
o The Bengal school of architecture was characterised by its use of bricks and black marble. The
mosques built during this period continued the use of sloping ‘Bangla roofs’, which was previously
used for temples.
o Examples: Qadam Rasul Mosque in Gour, Adina Mosque in Pandua, etc.

Q 2.C
• The Singhi Chham is a popular mask dance of Sikkim. In this group dance five boys led by a herdsman
dance like legendary Snowlion. The snowlion is a significant cultural symbol of the state. The dancers are
dressed in furry lion costumes symbolising the snow lion and pay tribute to Khang-Chen Dzong Pa
(Kanchenjunga Peak). Musical instruments (without any songs) used are Drum, Cymbal, Yarka and
Yangjey. Hence, option 1 is correct.
• Chhau dance: There are three main styles of Chhau dance – Saraikella Chhau in Jharkhand, Mayurbhanj
Chhau in Odisha and Purulia Chhau in West Bengal. Of these, Mayurbhanj Chhau artists do not wear
masks. Thematically, Chhau draws substantially from great epics like the Ramayan, the
Mahabharat and also from folk and tribal elements. The presiding deity of Mayurbhanj Chhau is
Lord “Bhairab”. Hence, option 2 is not correct.
• Bihu is a fast-paced, extremely joyful dance, hailing from the state of Assam. It is performed by
young girls and boys during the important agricultural festivals. The dance is performed to a twin-faced
drum, with one end played with a stick and the other with the palm. A Bihu performance is usually pretty
long and is full of exciting changes in rhythm, mood, movements, pace, tempo and improvisation. It does
not involve wearing of masks. Hence, option 3 is not correct.

1 www.visionias.in ©Vision IAS

Google it:- https://upscpdf.com


https://t.me/UPSC_PDF Download From > https://upscpdf.com https://t.me/UPSC_PDF

• Butta Bommalu literally means basket toys and is a popular dance form of the West Godavari district
of Andhra Pradesh. The dancers wear masks of different characters, resembling toy like shapes, and
entertain through delicate movements and non-verbal music. Hence, option 4 is correct.
• Padayani is a martial dance performed in the Bhagavati temples of Southern Kerala. Padayani
literally means rows of infantry, and it is a very rich and colourful affair. The dancers wear huge masks
known as kolams, and present interpretations of divine and semi divine narratives. It is performed to
worship Goddess Kali. Hence, option 5 is correct.

Q 3.B
• Statement 1 is not correct. Satavahanas show traces of matrilineal social structure. It was customary for
their king to be named after their mother (eg Gautamiputra Satakarni). Queens made important religious
gifts in their own right. But succession to the throne passed to the male member. So, the social structure
was only partially matrilineal.
• Statement 2 is not correct. They did not issue any Gold coin as the Kushans did. They mostly issued
coins of Lead, copper and bronze.
• Statement 3 is correct. Their official language was Prakrit. All inscriptions were composed in this
language and were written in Brahmi script.

Q 4.C
• Statement 1 is correct: Recently, the Defence Minister launched the Naval Innovation and Indigenisation
Organisation (NIIO). The NIIO puts in place dedicated structures for the end users to interact with
academia and industry towards fostering innovation and indigenisation for self-reliance in defence in
keeping with the vision of Atmanirbhar Bharat.
• Statement 2 is correct: The NIIO is a three-tiered organisation. Naval Technology Acceleration Council
(N-TAC) will bring together the twin aspects of innovation and indigenisation and provide apex level
directives. A working group under the N-TAC will implement the projects. A Technology Development
Acceleration Cell (TDAC) has also been created for induction of emerging disruptive technology in an
accelerated time frame.

Q 5.C
• Chaul and Dabhol were the major ports of the Bahmani kingdom in the Maharashtra region. It attracted
trading ships from the Persian Gulf and the Red Sea and poured in luxury goods from all parts of the
world. The earliest mention of Chaul is in the inscriptions at the Buddhist caves of Kanheri near
Mumbai. In the early years of the 14th century CE, Chaul fell to the invading army of Alauddin Khilji,
and after the fall of Deogiri, became a part of the Khilji Sultanate. Hence, pair 2 is not correctly
matched.
• Tamralipti or Tamralipta was a port city in the Bengal region, located on the Bay of Bengal in
Midnapore district of modern-day India in West Bengal. Tamralipta port has been an important port since
ancient times including the Chola period for traveling to Srilanka, Java, Sumatra, and Mayanmar through
different trade routes. Tamralipta port emerged as an important center of trade and commerce. However,
its importance declined with the loss of navigability of the Rupnarayan River. Hence, pair 1 is not
correctly matched.
• The town of Masulipatnam or Machlipatnam lay on the delta of the Krishna on Andhra coast. By the
late 15th century, Masulipatnam and its surrounding region became a battleground between the Bahamani
empire, the Vijayanagara empire and the kings of Odisha. A new and golden phase of Machilipatnam’s
history would begin under Sultan Quli Qutub Shah of Golconda (r. 1512-1543), who captured and then
developed the port. It emerged as the hub of the international diamond trade. Hence, pair 3 is correctly
matched.

Q 6.D
Causes for the rise of Magadha:
• Geographical factors
o Magadha was located on the upper and lower parts of the Gangetic valley.
o It was located on the mainland route between west and east India.
o The area had fertile soil. It also received enough rainfall.
o Magadha was encircled by rivers on three sides, the Ganga, Son and Champa making the region
impregnable to enemies.

2 www.visionias.in ©Vision IAS

Google it:- https://upscpdf.com


https://t.me/UPSC_PDF Download From > https://upscpdf.com https://t.me/UPSC_PDF

o Both Rajgir and Pataliputra were located in strategic positions and Rajgir was encircled by 5
hills which made it impregnable. Hence option 3 is correct.
o Hence this provided a well-developed communication network. Hence option 2 is correct.
• Economic factors:
o Magadha had huge copper and iron deposits.
o Because of its location, it could easily control trade.
o Had a large population which could be used for agriculture, mining, building cities and in the
army. Hence option 1 is correct.
• Cultural factors:
o Magadhan society had an unorthodox character.
o It had a good mix of Aryan and non-Aryan peoples.
o The emergence of Jainism and Buddhism led to a revolution in terms of philosophy and thought. They
enhanced liberal traditions.
o Society was not so much dominated by the Brahmanas and many kings of Magadha were ‘low’ in
origins.
• Political factors
o Magadha was lucky to have many powerful and ambitious rulers.
o They had strong standing armies.
o Availability of iron enabled them to develop advanced weaponry.
o They were also the first kings to use elephants in the army. Hence option 4 is correct.
o The major kings also developed a good administrative system.

Q 7.B
• Karkhana a manufacturing centre under state supervision during the Sultanate and Mughal periods and
now a common term for a place of manufacture or assembling. In view of the rural setting and subsistence
economy of India, the Muslim rulers, who came from a different background of production and
marketing, felt the need for maintaining state-sponsored and state-controlled karkhanas to supply the
royal household and departments of government with provisions, stores and equipment. Muhammad
bin Tughlaq is said to have established many such karkhanas. His successor Sultan Firuz Shah Tughlaq
turned these karkhanas into centres of vocational training as well. Many of the war captives, who were
turned slaves, were distributed among these karkhanas to be trained in manufacturing, arts and
crafts. Hence, statement 1 is not correct
• The Mughal emperors took special interest in the Karkhanas and saw to it that state maintained Karkhanas
were set up not only in the capital but also in the provincial headquarters as well as in other important
industrial towns. Hence, statement 2 is correct.
• At the time of Akbar karkhanas formed a regular department under Mir Saman (Khan-i-Saman). During
his time, greater attention was given to the training of apprentices in these karkhanas. These young
apprentices were placed under an Ustad (master craftsman) to learn the art and become experts themselves
in course of time. Hence, statement 3 is correct
Q 8.B
• Kalpa sutra is a Jain ancient text containing the biographies of the last two Jain Tirthankaras,
Parshvanath and Mahavira.
• It contains detailed life histories with illustrations. Adinath (or Rishabh Dev) and Neminath are two other
Tirthankaras briefly mentioned in the text, with Adinath depicted in some of the illustrations.
• The book is considered to be written by Bhadrabahu I.
• Additional Information:
o Bhadrabahu, who is considered by the Jains as the last Sutra-kevali (the one who has memorized all
the scriptures), has written many important works of Jaina literature.
o Apart from the Niyuktis (contain philosophical discussions on matters such as the existence of the
soul, analysis of knowledge, and meaning etc.), he also wrote the Samhitas which is a book dealing
with legal cases.
• Hence option (b) is the correct answer.

Q 9.C
• The World Wide Fund for Nature (WWF India), in collaboration with the Bombay Natural History
Society (BNHS), launched the third edition of Dragonfly Festival 2020. This is part of the National
Dragonfly Festival organised by WWF India, Bombay Natural History Society and Indian
Dragonfly Society in association with National Biodiversity Board, UN Environmental Programme,
UN Development Programme and IUCN-Commission on Education and Communication (CEC).
3 www.visionias.in ©Vision IAS

Google it:- https://upscpdf.com


https://t.me/UPSC_PDF Download From > https://upscpdf.com https://t.me/UPSC_PDF

• Dragon Flies are used as an:


o indicator of aquatic ecosystem quality and biodiversity. Their sensitivity towards habitat quality
and their amphibious cycle makes them well suited to evaluate environmental changes.
o The dragonflies require clean water to thrive and their survival also requires healthy vegetation
to provide them with oxygen. This makes them the best indicator of oxygen levels in the aquatic
environment.
o Dragonflies play a key role in maintaining a healthy ecosystem and are crucial in controlling the
mosquito population. Thus, they help in preventing the spread of diseases like malaria, dengue etc.
o They are used in traditional medicine in Japan and China, and caught for food in Indonesia.
o As adults, dragonflies eat mosquitos, small flies, butterflies, bees, and other dragonflies.
• The International Union for Conservation of Nature, Species Survival Commission, Dragonfly
Specialist Group (IUCN SSC DSG) is a global network of 50 scientists and conservationists from 28
countries, working on dragonflies and dragonfly related issues. They work on dragonfly related issues
in Arabian Peninsula, East Himalayas, Western Ghats, Arabian Peninsula, Indo-Burma, Africa, New
Zealand.

Q 10.C
• Researchers have shed new light on the dwarf planet Ceres, which lies in the asteroid belt between Mars
and Jupiter and is also the largest object in that belt. Ceres now has the status of an “ocean world”, after
scientists analysed data collected by NASA’s Dawn spacecraft.
• There are officially five dwarf planets in our Solar System. The most famous is Pluto, downgraded from
the status of a planet in 2006. The other four, in order of size, are Eris, Makemake, Haumea and Ceres.
The sixth claimant for a dwarf planet is Hygiea, which so far has been taken to be an asteroid.
• Last year, using observations made through the European Space Organisation’s SPHERE instrument at
the Very Large Telescope (VLT), astronomers found that Hygiea may possibly be a dwarf planet since it
satisfied the four criteria set by the International Astronomical Union (IAU) for a celestial body to
be called a dwarf planet.
• These four criteria are – that the body orbits around the Sun, it is not a moon, has not cleared the
neighbourhood around its orbit (which means it is not the dominant body in its orbit around the
Sun and this is what differentiates a planet from a dwarf planet) and has enough mass for its
gravity to pull it into a roughly spherical shape.
• Hence option (c) is the correct answer.

Q 11.B
• Among those who exercised power in the empire were military chiefs who usually controlled forts
and had armed supporters.
• These chiefs often moved from one area to another, and in many cases were accompanied by peasants
looking for fertile land on which to settle.
• These chiefs were known as Nayaks and they usually spoke Telugu or Kannada.
• Many nayakas submitted to the authority of the kings of Vijayanagara but they often rebelled and had to
be subdued by military action.
• The Amara-nayaka system was a major political innovation of the Vijayanagara Empire. It is likely that
many features of this system were derived from the iqta system of the Delhi Sultanate.
• The Amara-nayakas were military commanders who were given territories to govern by the Raya.
o They collected taxes and other dues from peasants, craftspersons, and traders in the area. They
retained part of the revenue for personal use and for maintaining a stipulated contingent of
horses and elephants. Hence statement 1 is correct.
o These contingents provided the Vijayanagara kings with an effective fighting force with which they
brought the entire southern peninsula under their control. Some of the revenue was also used for the
maintenance of temples and irrigation works.
o The amara-nayakas sent tribute to the king annually and personally, appeared in the royal
court with gifts to express their loyalty. Kings occasionally asserted their control over them by
transferring them from one place to another.
• However, during the course of the seventeenth century, many of these nayakas established
independent kingdoms. This hastened the collapse of the central imperial structure.
• The Chola traditions of village self-government were considerably weakened under the
Vijayanagara rule. The growth of hereditary nayakships tended to curb their freedom and
initiatives. Hence statement 2 is not correct and statement 3 is correct.
• Nayakas also contributed to the architecture and made several palaces and temples.
4 www.visionias.in ©Vision IAS

Google it:- https://upscpdf.com


https://t.me/UPSC_PDF Download From > https://upscpdf.com https://t.me/UPSC_PDF

Q 12.D
• Brahmadeya was tax free land gift either in form of single plot or whole villages donated to Brahmans. It
was initially practiced by the ruling dynasties and was soon followed up by the chiefs, merchants,
feudatories, etc. Brahmadeya was devised by the Brahmanical texts as the surest mean to achieve merit
and destroy sin.
• Brahmadeya helped the expansion of agrarian economy and the emergence of urban settlements. It also
helped the Kings to gain the ideological support for their rule. Brahamdeya sometimes also resulted in
alienation of peasant land rights and created social tension and clash leading even to death between
peasant, Brahmans and the Kings. Hence option (d) is correct.

Q 13.D
• The major expansion of the Sultanate took place during the reigns of Ghiyasuddin Balban, Alauddin
Khalji and Muhammad Tughluq. The campaigns of expansion were along the internal frontier and
external frontier.
o Campaigns along the internal frontier: These aimed at consolidating the garrison towns of hinterlands.
These included clearing of forests in the Ganga-Yamuna doab and expulsion of hunter-gatherers and
pastoralists from their habitats. These lands were given to peasants to encouraging agriculture. The
trade routes were protected and regional trade was promoted by establishing new fortresses and
towns.
o Campaigns along the external frontier: Military expeditions into southern India that started during the
reign of Alauddin Khalji culminated during the reign of Muhammad Tughluq. In their campaigns;
elephants, horses and slaves were captured and precious metals were plundered by the Sultanate
armies.
• Regarding the control of Malwa and Gujarat, the Turkish rulers had strong reasons for coveting this
region:
o Not only these areas were fertile and populous, but they also controlled the western seaports and trade
routes connecting them with the Ganga valley.
o The overseas trade from Gujrat ports brought in a lot of gold and silver wealth which has been
accumulated by the local rulers of the area.
o Another reason for the sultans of Delhi to establish their rule over Gujarat was that it could secure
them better control over the supply of horses for their armies. With the rise of Mongols in Central
and West Asia and their struggle with the rulers of Delhi, the supply of horses of good quality to
Delhi from this region had been beset with difficulties.
• Hence, option (d) is the correct answer.

Q 14.B
• Maand is a folk music from the State of Rajasthan. It is said to have developed in the royal courts and
hence is also recognised in the classical circles. The songs are usually about the bards singing the glory of
the Rajput rulers. It is considered close to Thumri or Ghazal. Hence pair 1 is not correctly matched.
• Khongjom Parba is an important folk music from the State of Manipur. It is a popular ballad genre
which is a musical narration of the battle of Khongjom fought between the British army and the Manipuri
resistance forces in 1891. Hence pair 2 is correctly matched.
• Wanawan is the folk music from Jammu and Kashmir. It is specially sung during wedding ceremonies
and is considered auspicious. Hence pair 3 is correctly matched.
• Baul is not only a type of music, but a Bengali religious sect. The music of the Bauls, Baul Sangeet, is
a particular type of folk song. Songs mainly belong to either Vaishnava Hindu or Sufi Muslim
sects. The prominent exponents of this music are: Yotin Das, Purno Chandra Das, Lalon Fakir, Naboni
Das and Sanatan Das Thakur Baul. Hence pair 4 is not correctly matched.

Q 15.D
Buddhist Councils:
• Buddhist Councils marked important turning points in early Buddhism.
• These councils resulted in sectarian clashes and the eventual Great Schism that resulted in the two major
schools, Theravada and Mahayana.
• In total, 4 major Buddhist councils were convened:

5 www.visionias.in ©Vision IAS

Google it:- https://upscpdf.com


https://t.me/UPSC_PDF Download From > https://upscpdf.com https://t.me/UPSC_PDF

First Council:
• It was held soon after the Mahaparinirvan of the Buddha, around 483 BC under the patronage of King
Ajatshatru and was presided by Mahakasyapa, a monk.
• The council was held in the Sattapani cave at Rajgriha.
• The council was held with the purpose of preserving Buddha’s teachings (Sutta) and rules for disciples.
During this council, the teachings of Buddha were divided into three Pitakas.
Second Council:
• It was held in Vaishali, a village in Bihar under the patronage of the king Kalasoka in 383 BC. It was
presided by Sabakami.
Third Council:
• It was held in 250 BC in Patliputra under the patronage of Ashoka and was presided by Moggaliputta
Tissa.
• Fourth Council:
• It was held in 72 AD at Kundalvana, Kashmir. It was presided by Vasumitra, while Asvaghosa was his
deputy under the patronage of King Kanishka of Kushan Empire.
• Buddhism was divided into two sects namely Mahayana and Hinayana. Hence option (d) is the correct
answer.

Q 16.C
• Rajendra Chola I or Rajendra I was a Tamil Chola emperor of South India.
• He succeeded his father Rajaraja Chola I to the throne in 1016 CE. During his reign, he extended the
influence of the Chola empire to the banks of the river Ganga in North India and across the Indian Ocean
to the West and South East Asia, making the Chola Empire one of the most powerful maritime empires of
India.
• He defeated Mahipala, the Pala king of Gauda in present-day Bengal and Bihar along with another local
king.
• To commemorate his victory he assumed the title of 'Gangaikondachola' (Chola who conquered the
kingdoms near Ganga).
• He also built a new capital city called Gangaikonda Cholapuram on the banks of river Kaveri.
• Hence, the option (c) is the correct answer.

Q 17.B
• The Samkhya and Vaisheshika systems put forward the materialist view of life. Kapila, the earliest
exponent of the Samkhya, teaches that a man's life is shaped by the forces of nature and not by any divine
agency. The Vaisheshika school gave importance to the discussion of material elements or dravya.
• Materialistic ideas also appear in the doctrines of the Ajivikas. But Charvaka was the main expounder
of the materialistic philosophy. This philosophy came to be known as the Lokayata, which means the
ideas derived from the common people. He showed lack of belief in the other world and was opposed to
the quest for spiritual salvation.
• Mimamsa literally means the art of reasoning and interpretation. But reasoning was used to justify
various vedic rituals and emphasized that vedas contain eternal truth. Hence, it was spiritualistic in
nature. Hence, the correct option is B.

Q 18.B
• The Mauryas made a remarkable contribution to art and architecture.
• They introduced stone masonry on a wide scale. hence statement 3 is correct
• The remains certainly attest the high technical skill attained by the Maurya artisans in polishing the stone
pillars.
• Each pillar is made of a single piece of buff coloured sandstone. Only their capitals, which are
beautiful pieces of sculpture in the form of lions or bulls, are joined with the pillars on the top.
These polished pillars were set up throughout the country, which shows that technical knowledge
involved in their polishing and transport had spread far and wide. Hence statement 2 is correct.
• Marble was not prevalent during the Mauryan times. hence statement 1 is not correct.
• The Maurya artisans also started the practice of hewing out caves from rocks for monks to live in. The
earliest examples are the Barabar caves at a distance of 30 km from Gaya.
• In the Maurya period, burnt bricks were used for the first time in north-eastern India.

6 www.visionias.in ©Vision IAS

Google it:- https://upscpdf.com


https://t.me/UPSC_PDF Download From > https://upscpdf.com https://t.me/UPSC_PDF

• New material culture in the Gangetic region was based on an Intensive use of iron, the prevalence of
writing, plenty of iron implements.
• This period shows socket axes, sickles and Closures low shares. The spoked wheel also came to be used.
Q 19.D
• The Jain sacred literatures were initially preserved orally from the time of Mahavira. These literatures
were systematized by the Jain council from time to time. The first systematization of the Jain canonical
literature took place in a council at Pataliputra (Patna) by the end of 4th Century B.C. and again in two
other councils in the early 3rd century B.C. in Mathura and Valabhi. The fourth and last Jain council took
place at Valabhi in 454 or 467 4.D. This council is said to be the source of the Svetambara Jain scripture.
The Svetambara canon consists of 45 Agamas: 11 Anga (parts) (originally consisted of 12 Angas, the 12th
having been lost) 12 Upanga (sub-parts) 4 Mula-Sutra, 6 Chedasutras, 2 Chulika-sutras, 10 Prakirnakas
(mixed texts). This makes the canon of the lain (Svetambara) religion.
• Thus the Svetambaras follow the Agama as their secret scripture. The Digambaras, however, are of the
opinion that the original canon of Jainism is lost and that the substance of Mahaviras' message is
contained in the writings of ancient religious figures. They recognise two works in Prakrit: the
Karmaprabhrita - chapters on Karman composed by Puspandanta and Bhutabalin and the
Kashayaprabhrita - chapters on Kasayas composed by Gunudhara. Hence, option (d) is the correct
answer.
Q 20.A
• The UAE has launched Agriota, a new technology-driven agri-commodity trading and sourcing e-
market platform that will bridge the gap between millions of rural farmers in India and the Gulf
nation's food industry. Under the initiative, launched earlier this week by the Dubai's free-zone Dubai
Multi Commodities Centre (DMCC) and the government of Dubai's authority on commodities trade and
enterprise, millions of Indian farmers will get an opportunity to connect directly with the entire food
industry in the UAE, including food processing companies, traders and wholesalers through the Agriota-E
Marketplace platform. Hence option (a) is the correct answer.
• The marketplace allows the farmers to bypass intermediaries, optimising the supply chain and ensuring
traceability to create value for all stakeholders. The online marketplace also provides end-to-end
traceability and transparency through last-mile verification and extension infrastructure in a blockchain
environment.
Q 21.A
• Firoz Shah Tughlaq (1309-1388) was one of the rulers of the Tughlaq dynasty, who reigned over the
Sultanate of Delhi from 1351 to 1388.
• He succeeded his cousin Muhammad bin Tughlaq following the latter's death at Thatta in Sindh.
• His autobiography, Futuhat-e-firozshahi is one of the major sources of information about his reign.
• Some of his major contributions can be noted as follows:
o He instituted various policies to increase the material welfare of his people.
o Many rest houses (sarai), gardens and tombs were built for travellers and traders.
o A number of madrasas (Islamic religious schools) were opened to encourage the religious education
of Muslims.
o He commissioned many public buildings in Delhi.
o He built five major canals to bring more land under cultivation.
o He set up hospitals for the free treatment of the poor and encouraged physicians in the development of
Unani medicine.
o He founded several cities around Delhi, including Jaunpur, Firozpur, Hissar, Firozabad, Fatehabad.
o He provided money for the marriage of girls belonging to poor families under the department of
Diwan-i-khairat.
o He banned inhuman punishments such as cutting of nose, hands, feet for petty offenses like
theft. Hence, statement 3 is not correct.
o He was the first ruler to get the Hindu religious translated from Sanskrit to Persian and Arabic so that
there may be a better understanding of Hindu ideas and practices. Hence, statement 2 is not correct.
o Many books on music, medicine and mathematics were also translated from Sanskrit into Persian
during his reign. He had a large personal library of manuscripts in Persian, Arabic and other
languages
• However, he was a fervent Muslim and adopted strict sharia policies. He imposed Jizya tax on all non-
Muslims. Jizya became a separate tax. Earlier it was part of land revenue. Hence, statement 1 is correct.
• He refused to exempt the Brahmans from payment of Jizyah since this was not provided in the Sharia.
• He persecuted a number of Muslim sects which were considered heretical by the theologians.

7 www.visionias.in ©Vision IAS

Google it:- https://upscpdf.com


https://t.me/UPSC_PDF Download From > https://upscpdf.com https://t.me/UPSC_PDF

Q 22.C
• Turks were responsible for establishing the Muslim rule in India. The Turks were new converts to Islam
and therefore, proved more fanatical in their religious zeal as compared to the Persians and the Arabs.
• With a belief in the superiority of their race, inspired by their new religion, determined to propagate Islam
and relying on the strength of their arms, the Turks conquered a large part of western Asia and, ultimately,
moving towards the east penetrated into India.
• Sultan Mahmud of Ghazni was the first to penetrate deep into India. However, it was the Muhammad of
Ghur who actually established the Muslim empire in India after the decisive second battle of Traian in
1192.
• Several factors which were responsible for the defeat of Indian defense and subsequently facilitated the
Turkish invasion are as follows:
o Politically, India was divided into many kingdoms which constantly fought against each other.
Because of their internal conflicts, none of them could utilize its complete resources, nor could they
unite themselves against foreign invasions. Hence option 1 is correct.
o Socially, the division of the Hindus into castes and sub-castes had created sharp differences between
sections of the society and therefore, had weakened it.
o After the defeat of Muhammad Ghuri in the first battle of Tarain in 1191, Prithviraj Chauhan, the
then ruler in Northern India was contended with the victory and did not completely oust the opponent.
This provided them with an opportunity to regroup his forces and make another bid for India next
year.
o The Turkish army was better organized and well-led by able leaders. The superior organization
skills and speed of movement of the Turkish cavalry and their mounted archers dominated the
numerically strong and equally equipped Indian forces. Hence option 2 is not correct and 3 is
correct.
o Various local rulers, especially the ruler of Kannuaj, Jaichandra possessed a strong army but did not
choose to support the India cause in the battle.
• Thus, the Hindushahi kingdom failed to check the growing power of the Turks towards the east.

Q 23.D
• Both Jainism and Buddhism admitted women into the sanghas. Hence statement 1 is not correct.
• Jainism does not condemn the varna system while Buddhism does.
• Jainism believed in the transmigration of the soul i.e. reincarnation while Buddhism does not.
• Buddhism prescribes a middle path while Jainism advocates its followers to live a life of complete
austerity. Hence statement 2 is not correct.
• Non-violence cannot be compromised in Jainism whereas in Buddhism violence is inevitable. Jains
believe that the only way to save one's own soul is to protect every other soul, and so the most central Jain
teaching, and the heart of Jain ethics, is that of ahimsa (non-violence).
Q 24.D
• Recently, Arunachal Pradesh assembly passes resolution to bring state under Sixth Schedule. The
Arunachal Pradesh Assembly has passed a resolution to bring the state under the ambit of Sixth Schedule
of the Constitution besides amending Article 371(H) bringing provisions to safeguard tribes of the
state. Hence option (d) is the correct answer.
• About Sixth Schedule: The Sixth Schedule consists of provisions for the administration of tribal areas
in Assam, Meghalaya, Tripura and Mizoram, according to article 244 of the Indian Constitution. It
seeks to safeguard the rights of the tribal population through the formation of Autonomous District
Councils (ADC).

Q 25.B
• The twelfth-century witnessed the emergence of a new movement in Karnataka, led by a
Brahmana named Basavanna (1106-68) who was a minister in the court of a Kalachuri ruler. His
followers were known as Virashaivas (heroes of Shiva) or Lingayats (wearers of the linga). Hence
statement 1 is not correct.
• Lingayats continue to be an important community in the region to date. They worship Shiva in
his manifestation as a linga, and men usually wear a small linga in a silver case on a loop strung over
the left shoulder. Those who are revered include the jangama or wandering monks.
• Lingayats believe that on death the devotee will be united with Shiva and will not return to this world.
Therefore they do not practice funerary rites such as cremation, prescribed in the Dharmashastras. Instead,
they ceremonially bury their dead.

8 www.visionias.in ©Vision IAS

Google it:- https://upscpdf.com


https://t.me/UPSC_PDF Download From > https://upscpdf.com https://t.me/UPSC_PDF

• The Lingayats challenged the idea of caste and the “pollution” attributed to certain groups
by Brahmanas. They also questioned the theory of rebirth. These won them, followers, amongst
those who were marginalized within the Brahmanical social order. Hence statement 2 is correct.
• The Lingayats also encouraged certain practices disapproved in the Dharmashastras, such as post-puberty
marriage and the remarriage of widows.
• Our understanding of the Virashaiva tradition is derived from vachanas (literally, sayings) composed
in Kannada by women and men who joined the movement. Hence statement 3 is not correct.
Q 26.B
• Sallekhana or santhara is a controversial religious practice in Jainism.
• In it followers stop eating with the intention of preparing for death. It is seen as the ultimate way to
expunge all sins and karma, liberating the soul from the cycle of birth, death and rebirth. . It is a ritual
fast to death which Jains regard as the "culmination of our life as ascetics”.
• Hence option (b) is the correct answer.

Q 27.A
• Recently it has been reported that a dent present on the protective magnetic shield over the surface
of Earth is widening. The dent, which has been present over South America and the South Atlantic
Ocean for several decades, is known as the South Atlantic Anomaly (SAA). Nasa has said the
anomaly is now growing and although it will not have an impact on Earth, it can throw navigation in
space out of gear. Hence option (a) is the correct answer.
• The magnetic field on which the dent has occurred protects the Earth from particle radiation of the Sun.
With the South Atlantic Anomaly widening, the weak spot is letting the harmful particle radiation from
Sun come near to the surface. Not only that, this anomaly also affects satellites that pass through the area
and tamper with the navigation.
• Nasa’s Earth scientists are now monitoring the changes to predict future changes and prepare for the
future challenges this weak spot in the Earth’s magnetic field may throw at space missions and
satellites. According to a report in The Business Insider, the European Space Agency (ESA) had said in
May that the magnetic field has lost 9% strength over the past 200 years. The weak spot in this shield has
further worsened 8% since 1970. Now, Nasa has said that the dent is splitting into two lobes.
• Nasa has said the South Atlantic Anomaly (SAA) is caused by processes in the Earth. The tilt of the
magnetic axis and the molten core of the Earth both cause the dent in the magnetic field. Forces in the
Earth’s core and the tilt in the magnetic axis together cause the anomaly, leading to the weak spot in the
protective magnetic shield.

Q 28.A
• The advent of Purandaradas (1484-1565) marked a very important landmark in the development of
Carnatic music has been justly termed as "Carnatic Sangeeta Pitamaha". Hence, statement 1 is
correct.
• He belonged to the Vaishnava tradition during the Vijayanagar rule. Prior to the adoption of Vaishnava
tradition, he was a rich merchant and was called Srinivasa Nayaka.
• The proponents of the theory that Puranadara Dasa was born in Malnad, refer to his older name and point
out that Nayaka title was attributed to locally influential people, including wealthy merchants in Malnad
during the Vijayanagar rule.
• He was a great devotee of Lord Krishna, a poet and a musician.
• He identified 84 ragas and devised a system of teaching Carnatic music in graded lessons.
• He introduced the Malavagowla scale as the basic scale for music instruction.
• He composed songs in Kannada and Sanskrit with the pen name Purandara Vithala. In his
compositions, he expressed his love for Sri Krishna describing various aspects of Krishna’s life.
• His kirtanas are popularly referred to as Dasara Padas or Devarnamas. Hence, statement 2 is not
correct.

Q 29.D
• From the archaeological finds of the Indus Valley Civilization it appears that the people of the Indus
Valley were conscious of fashion. Different hairstyles were in vogue and wearing (cut in different styles)
of beard was popular among all. Cosmetic products like cinnabar, face paint, lipstick and collyrium
(eyeliner) were also in use.
• Bead factories have been discovered at Chanhudaro and Lothal. Both being coastal cities getting
beads from neighbouring waters was easy and had dedicated bead-jewellery making

9 www.visionias.in ©Vision IAS

Google it:- https://upscpdf.com


https://t.me/UPSC_PDF Download From > https://upscpdf.com https://t.me/UPSC_PDF

industries. Beads were made of cornelian, amethyst, jasper, crystal, quartz, steatite, turquoise, lapis lazuli,
etc. Hence, statement 1 is not correct.
• The Harappan men and women decorated themselves with a large variety of ornaments produced from
every conceivable material ranging from precious metals and gemstones to bone and baked clay. They
were so much fond of jewellery that a cemetery has been found at Farmana in Haryana where dead
bodies were buried with ornaments, may be for use in the other world. Hence, statement 2 is not
correct.

Q 30.B
• The second Sangam was held at Kapadapuram but the all the literary works had perished except
Tolkappiyam.
• Tolkappiyam authored by Tolkappiyar is the earliest of the Tamil literature. It is a work on Tamil
grammar but it provides information on the political and socio-economic conditions of the Sangam period.
• The compositions of the third Sangam are classified into eight anthologies known
as Ettuttokoi/Ettuthokai and ten idylls known as Pattuppattu
• Silappadikaram Composed by Ilango Adigal. The story revolves around an anklet. The name literally
means the tale of an anklet. A chief character is Kannagi, who seeks revenge on the Pandya kingdom for
her husband who was wrongly put to death. The poem gives a lot of insight into contemporary Tamil
society, polity, values and social life of the people.
• Pathinenkilkanakku contains eighteen works about ethics and morals. The most important among
these works is Tirukkural authored by Thiruvalluvar, the tamil great poet and philosopher.
• Hence option (b) is the correct answer.

Q 31.D
• There were hardly any changes in the structure of Hindu society during the period. The society continued
to be divided on caste identities.
• There were little changes in the position of women in Hindu society. Old rules governed the marriages,
and the wife's obligation of service and devotion to husband continued. The Practice of sati was prevalent,
especially among higher caste society.
• As per the records of Ibn Battuta, widows were given the right to the property of a sonless husband,
provided the property was not joint i.e. had been divided. Hence, statement 1 is not correct.
• They not only have the right guardianship right to property but also had the right to dispose it. Thus it can
be said that property rights of women improved in the Hindu Law.
• In the early phase of the Delhi sultanate, a number of Hindu temples were sacked and converted to
mosques. The policy towards temples and places of worship of the Hindus, Jains etc. rested on the Sharia
Law which forbade the new places of worship being built in opposition to Islam. There was no ban on
erecting temples in villages since there were no practices of Islam there. Similarly, temples could be built
within the private space of homes. Hence, statement 3 is not correct.
• Despite the significant influence of orthodox theologians, a policy of broad religious toleration was
maintained. Conversion to Islam was not effected by the strength of the sword. Even, the Chaitanya guru
converted a number of Muslims as a part of the reformation. Conversion to Islam was due to the hopes of
political or economic gains or to improve one/s social position. Hence, statement 2 is not correct.

Q 32.A
• Some of the Chalcolithic cultures are:
o 2100–1500: Ahar Chalcolithic culture (Banas valley, Rajasthan). Their distinctive pottery feature is
black and red ware.
o 2000–1800: Kayatha Chalcolithic culture (MP). The sturdy red slipped ware with chocolate designs is
the main feature.
o 1400-750: Jorwe culture (Maharashtra). Jorwe ware is painted black-on-red and has a matt surface
treated with a red wash.
• Hence option (a) is the correct answer.

Q 33.B
• Pair 1 is correctly matched: Recently, a powerful typhoon was blowing over Japan’s southernmost
islands on course for Japan’s main southern island. The Japanese Meteorological Agency said Typhoon
Maysak was expected to later make landfall on Kyushu, Japan’s southern main island, bringing strong
winds and rain, then hit South Korea.

10 www.visionias.in ©Vision IAS

Google it:- https://upscpdf.com


https://t.me/UPSC_PDF Download From > https://upscpdf.com https://t.me/UPSC_PDF

• Pair 2 is not correctly matched: Recently cases of Covid-19 have been recorded among the Great
Andamanese tribe, setting off alarm bells in the Andaman and Nicobar Islands administration. A
Particularly Vulnerable Tribal Group (PVTG), the Great Andamanese now number only 59. The new
cases have been discovered in the remote Strait Island where the tribe is based. Strait Island is thus
located in India.
• Pair 3 is correctly matched: Recently Turkey and Greece are locked in a tense standoff over claims to
hydrocarbon resources in the eastern Mediterranean Sea. Both countries’ forces have been conducting
military exercises in the seas between Cyprus and the Greek island of Crete. The standoff was sparked
when Turkey sent its research vessel, Oruc Reis, accompanied by warships to search for gas and oil
reserves. Turkey disputes Greece’s claims that waters, where it’s searching for hydrocarbons, are part of
Greece’s continental shelf, insisting that Greek islands near Turkey’s coast cannot be taken into account
when delineating maritime boundaries. Ankara accuses Athens of trying to grab an unfair share of the
eastern Mediterranean’s resources.

Q 34.D
• Statement 1 is correct: Indian exports by the traders mainly included saltpeter, sugar, rice, spices, and
textiles. Portuguese engaged themselves in trade in these items through the sea route. They also tried to
control sea routes and provided safeguards to other nationals against sea pirates.
• Statement 2 is correct: Portuguese traders also helped in transmitting potato, tobacco, maize, cocoa, and
peanuts from Central America to India. But Portuguese were hardly able to change the established pattern
of Asian trade networks. The Gujarati and Arab traders continued to dominate the most lucrative Asian
trade like gold and horses from West Asia, and silk and porcelain from China.
• Statement 3 is correct: As a policy measure, trade in pepper, arms, and ammunition and war horses in
India was a royal monopoly and no private traders were allowed to engage in the trade of these goods.
Even though the Portuguese tried to change this but due to the presence of a strong Vijayanagara empire,
they failed to achieve success.

Q 35.B
• Lal Ded, a Kashmiri Muslim poetess, gave a new dimension to mystical bhakti. Lal Ded & Habba Khatun
represented the Sant tradition of bhakti and wrote Vakhs (maxims), which are peerless gems of spiritual
experience.
• Lal Ded, as a poet, was primarily Shaiva in orientation. She was a follower and a preacher of the Shaiva
philosophy of thought. Hence statement 1 is not correct.
• Her poems carry the mystic, philosophical, social, religious and womanly resonances. She aimed at
awakening the latent faculties of the soul in order to facilitate its leap into the Ultimate reality.
• Lal Ded is known as a pioneer of Kashmiri literature. She uttered her verses in Kashmiri and they are
also known as Vakhs or Lallavakhs. Hence statement 2 is correct.
• Vakhs are poems in the form of four lines. Her Vakhs are the outpourings of her soul, the expressions of
her inner experience and also the comments on social situations around her

Q 36.A
• The most magnificent building of the 13th century was the Qutub Minar which was founded by Aibek
and completed by Iltutmish. Hence statement 1 is correct.
• This seventy-one-meter tower was dedicated to the Sufi saint Qutbuddin Bakthiyar Kaki. The balconies of
this tower were projected from the main building and it was proof of the architectural skills of that period.
Later, Alauddin Khalji added an entrance to the Qutub Minar called Alai Darwaza. The dome of this arch
was built on scientific lines
• Amir Khusrau (1253–1325) was a contemporary of Balban, Jalaluddin Khilji and Alauddin Khilji.
He introduced many new ragas such as Ghora and Sanam. He evolved a new style of light music
known as Qawwalis by blending the Hindu and Iranian systems. The invention of the sitar was also
attributed to him. Hence statement 2 is not correct.
• Amir Khusrau (1252-1325) was the famous Persian writer of this period. He wrote a number of poems.
He experimented with several poetical forms and created a new style of Persian poetry called Sabaqi-Hind
or the Indian style.
• Amir Khusrau’s Khazain-ul-Futuh speaks about Alauddin Khilji’s conquests. His famous work
Tughlaq Nama deals with the rise of Ghiasuddin Tughlaq or Ghazi Malik. Hence statement 3 is not
correct.

11 www.visionias.in ©Vision IAS

Google it:- https://upscpdf.com


https://t.me/UPSC_PDF Download From > https://upscpdf.com https://t.me/UPSC_PDF

Q 37.B
• The Parthians were followed by the Kushans, who are also called Yuechis or Tocharians.
• Kushanas were of nomadic origins living near the steppes of north-central Asia near China.
• Due to the great wall, these were driven towards the Hindukush region.
• The Kushanas were credited with the defeat of the last king of the Greeks in Kabul.
• Their Empire extended from the Oxus to the Ganga, from Khorasan in Central Asia to Varanasi in Uttar
Pradesh. Kushans had two successive dynasties.
• The first dynasty was founded by a house of chiefs who were called Kadphises and who ruled for 28
years.
• The first was Kujula Kadphisesor Kadphises-I and the second king was Wima Kadphises or Kadphises II.
He issued a large number of gold coins with high-sounding titles like the ‘Lord of the Whole
World’. Hence, statement 1 is not correct.
• The Indo-Greeks were the first rulers in India to issue coins which can definitely be attributed to
the kings. They were the first to issue gold coins in India.
• The early Kushan kings issued numerous gold coins with a higher degree of metallic purity than is
found in the Gupta gold coins. Hence, statement 3 is not correct.
• Kushan coins, inscriptions, constructions and pieces of sculpture found in Mathura show that it was
their second capital in India, the first being Purushapura or Peshawar.
• Kanishka was the most famous Kushan ruler. He started an era in A.D. 78, which is now known as
the Shaka era and is used by the Government of India.
• secondly, Kanishka extended his wholehearted patronage to Buddhism. He held a Buddhist council
in Kashmir, where the doctrines of the Mahayana form of Buddhism were finalized. Kanishka was
also a great patron of art and Sanskrit literature.
• The Kushan rulers worshipped both Shiva and the Buddha, and the images of these two gods appeared
on the Kushan coins. Hence, statement 2 is correct.
• Several later Kushan rulers were worshippers of Vishnu. Some of them bore typical Indian names such
as Vasudeva.

Q 38.A
• The principal traits of Aryan culture are set out by Vedic, Iranian, and Greek literary texts and cognate
terms found in the Proto-Indo-European languages. The texts that help us to reconstruct the material and
other aspects of Aryan culture comprise the Rig Veda, the Zend-Avesta, and Homer’s Iliad and Odyssey.
• The Rig Veda is assigned to roughly 1500 BC, although the later additions might be as late as 1000 BC.
The earliest parts of the Zend- Avesta are roughly attributed to 1400 BC, and Homer’s works are assigned
to 900–800 BC. Though these texts belong to different areas, they suggest the period when copper and
bronze were in use. The later portions of Homer also mention iron.
• Generally, the texts represent agriculture and pastoralism as the principal sources of livelihood. The
people lived in a temperate climate. They domesticated horses that were used for riding and for
driving carts. They used spoked wheels and fought with bows and arrows which were placed in
quivers. They lived in a male-dominated society.
• The cults of fire and soma prevailed among the speakers of the Indo-European languages in Iran
and the Indian subcontinent. However, animal sacrifice, including horse sacrifice, seems to have
been practiced by all the Indo-European communities.
• The use of birch-wood seems to be an Aryan feature along with underground houses. The birch is
called bhurja in Sanskrit, and it has cognates in six Indo-European languages.
• Hence all the statements are correct.

Q 39.A
• Alaud-Din Khalji (1296–1316) was born as Ali Gurshasp. He was the most powerful emperor of the
Khalji dynasty that ruled the Delhi Sultanate.
• He instituted a number of significant administrative changes, related to revenues, price controls, and
society. He is noted for repulsing the Mongol invasions of India.
• He undertook large-scale reforms to be able to maintain a large army and to weaken those capable of
organizing a revolt against him.
• Revenue reforms
o He took away all landed properties of his courtiers and nobles and canceled revenue assignments
which were henceforth controlled by the central authorities.
o He brought a large tract of fertile land under the directly-governed crown territory, by eliminating
iqta's, land grants and vassals in the Ganga-Yamuna Doab region.
12 www.visionias.in ©Vision IAS

Google it:- https://upscpdf.com


https://t.me/UPSC_PDF Download From > https://upscpdf.com https://t.me/UPSC_PDF

o He imposed a 50% kharaj tax on the agricultural produce in a substantial part of northern India to be
collected directly from the cultivators.
o He fixed the tax proportional to land area.
o He recruited many well paid accountants, collectors and agents, and account books were regularly
audited.
• Market reforms
o He fixed the prices for a wide range of goods, including grains, cloth, slaves and animals.
o His administration set up granaries and stocked them with grain collected from the peasants
o He banned hoarding and held the transporters, their agents and their families collectively responsible
for any violation
o He prescribed standards for weights and measures and a dedicated office of nazir was established
for this purpose.
• Military reforms
o He maintained a large standing army, including 475,000 horsemen according to the 16th-century
chronicler Firishta.
o He managed to raise such a large army by paying relatively low salaries to his soldiers and introduced
market price controls to ensure that the low salaries were acceptable to his soldiers.
o He maintained a descriptive roll (chehra) of every soldier and established a system of branding the
horses (dagh system).
• The token currency was introduced by Muhammad Bin Tuglaq. The token currency had inscriptions
marking the use of new coins instead of the royal seal and so the citizens could not distinguish between
the official and the forged coins.

Q 40.B
Ellora caves paintings:
• Number of Caves: It is a group of 34 caves – 17 Brahmanical, 12 Buddhist, and 5 Jain. These sets of
caves were developed during the period between the 5th and 11th centuries A.D. (newer as compared to
Ajanta Caves) by various guilds from Vidarbha, Karnataka and Tamil Nadu. Thus paintings painted
there are also newer as compared to Ajanta Paintings. Hence statement 1 is not correct.
• These paintings are related to all three religions (Hinduism, Buddhism, and Jainism). Hence
statement 2 is correct.
• The most important characteristic features of Ellora painting are the sharp twist of the head, painted
angular bents of the arms, the concave curve of the close limbs, the sharp projected nose, and the long-
drawn open eyes, which can very well be considered as the medieval character of Indian paintings.

Q 41.B
• The Prayaga Prashasti (also known as the Allahabad Pillar Inscription) was composed in Sanskrit by
Harisena mentioning the achievements of the Gupta ruler Samudragupta (c. fourth century CE). Apart
from the Prayag prashasti inscription, the Allahabad pillar also has inscriptions related to Mauryan
emperor Ashoka and Mughal emperor Jahangir.

Q 42.C
• Sangeet Natak Akademi has recognised eight classical dance forms includes Bharatnatyam,
Kuchipudi, Kathakali, Mohiniattam, Odissi, Manipuri, Kathak and Sattriya. Hence, statement 1 is
correct.
• Sattriya dance in modern-form was introduced by the Vaishnava Saint Sankaradeva in the 15th
century AD in Assam. Hence, statement 2 is correct.
• The art form derives its name from the Vaishnava monasteries known as ‘Sattras’, where it was primarily
practiced. It finds mention in the ancient text ‘Natya Shastra’ of sage Bharat Muni. It is inspired from the
Bhakti Movement.
• Some of the features of Sattriya dance include:
o The focus of the Sattriya recitals is to own the devotional aspect of dance and narrates mythological
stories of Vishnu. Hence, statement 3 is not correct.
o Sattriya dance also includes Nritta, Nritya and Natya.
o The dance is generally performed in groups by male monks known as ‘Bhokots’ as part of their
daily rituals or events on festivals. Hence, statement 4 is not correct.

13 www.visionias.in ©Vision IAS

Google it:- https://upscpdf.com


https://t.me/UPSC_PDF Download From > https://upscpdf.com https://t.me/UPSC_PDF

Q 43.B
• The Pala Empire was an imperial power during the post-classical period (8th and 9th century AD) which
originated in the Bengal region.
• The empire was founded with the election of Gopala as the emperor of Gauda in 750 CE.
• Arab merchant Suleiman (951 CE), referred to the Pala kingdom as Ruhma or Rahma.
• At its height in the early 9th century, the Pala Empire was the dominant power in the northern Indian
subcontinent, with its territory stretching across the Gangetic plain to include parts of modern-day eastern
Pakistan, northern and northeastern India, Nepal and Bangladesh.
• Pala control of North India was short-lived as they struggled with the Gurjara-Pratiharas and the
Rashtrakutas for the control of Kannauj and were subsequently defeated.
• They were followers of the Mahayana and Tantric schools of Buddhism.
• The Palas developed the Buddhist centers of learnings, such as the Vikramashila and the Nalanda
universities.
• Mahipala I ordered construction and repairs of several sacred structures at Saranath, Nalanda and Bodh
Gaya.
• Hence, option (b) is the correct answer.

Q 44.C
• Recent context: National Fertilizers Limited (NFL) a PSU under the Department of Fertilizers has
displayed Maharashtra’s famous Warli painting on the outer walls of its Corporate Office in Noida.
About Warli painting:
• The Warlis or Varlis are an indigenous tribe or Adivasis, living in mountainous as well as coastal areas of
the Maharashtra-Gujarat border and surrounding areas.
• Maharashtra is known for its Warli folk paintings.
• Its roots may be traced to as early as the 10th century A.D.
• They bear a resemblance to those created in the Rock Shelters of Bhimbetka.
• Scenes portraying hunting, fishing and farming, festivals and dances, trees and animals are used to
surround the central theme of the painting.
• All the paintings are done on a red ochre or dark background, while the shapes are white in colour.
• Tribal women are mainly engaged in the creation of these paintings.
• Unique features: A very basic graphic vocabulary like a circle, a triangle and a square are used in these
rudimentary wall paintings which are monosyllabic in nature.
• Pattachitra or Patachitra is a general term for traditional, cloth-based scroll painting, based in the
eastern Indian states of Odisha and West Bengal.
• Kalamkari is an ancient style of hand painting done on cotton or silk fabric with a tamarind pen, using
natural dyes. The word Kalamkari is derived from a Persian word where 'kalam' means pen and 'kari'
refers to craftsmanship. Kalamkari is a type of hand-painted or block-printed cotton textile, produced in
Isfahan, Iran and the Indian states of Andhra Pradesh and Telangana. Only natural dyes are used in
kalamkari, and it involves twenty-three steps.
• Gond paintings are a form of painting from folk and tribal art that is practiced by one of the largest tribes
in India with whom it shares its name. Gond comes from the Dravidian expression, Kond which means
'the green mountain'.
• Hence option (c) is correct.

Q 45.D
• Sufism is a mystical form of Islam, a school of practice that emphasizes the inward search for God and
shuns materialism.
• They were pious men who condemned the degeneration in political and religious life. They opposed the
vulgar display of wealth in public life and the readiness of the ulema to serve “ungodly” rulers.
• The Sufis laid emphasis upon free thought and liberal ideas. They were against formal worship, rigidity
and fanaticism in religion. The Sufis turned to meditation in order to achieve religious satisfaction. Like
the Bhakti saints, the Sufis too interpreted religion as ‘love of god’ and service of humanity.
• The Sufis organised samas (a recital of holy songs) to arouse mystical ecstasy.
• The advent of Sufism in India is said to be in the eleventh and twelfth centuries, around the phase of
Turkish Invasion of India and much before the Mughal empire. Hence, statement 1 is not correct.
• It must be noted that the Sufi saints were not setting up a new religion, but were preparing a more liberal
movement within the frameworkof Islam. Hence, statement 2 is not correct.
• They owed their allegiance to the Quran as much as the ulema did. Hence, statement 3 is not correct.
14 www.visionias.in ©Vision IAS

Google it:- https://upscpdf.com


https://t.me/UPSC_PDF Download From > https://upscpdf.com https://t.me/UPSC_PDF

• In course of time, the Sufis were divided into different silsilahs (orders) with each silsilah having its own
pir (guide) called Khwaja or Sheikh. The pir and his disciples lived in a khanqah (hospice). A pir
nominated a successor or wali from his disciples to carry on his work.

Q 46.C
• Bhakti movement was a revolution started by the Hindu saints to bring religious reforms by adopting the
method of devotion to achieve salvation. This movement resulted in various rites by practicing rituals of
devotion among the Hindus, Muslims and Sikhs in the Indian subcontinent.
• It was initiated by Alvars and Nayayars in Souther India and gradually led by various other saints in
different parts of India.
• Bhakti Movement in North India
o Ramananda was the most prominent scholar saint of the Vaishnava bhakti in Northern India. He is
considered to be the link between the south Indian bhakti tradition and North Indian Vaishnava
bhakti.
o Another prominent saint was Vallabhacharya. He was the founder of Pushtimarga, also known as
Vallabha sampradaya (Vallabha Sect).
o Tulsidas (1532-1623) championed the cause of Rama bhakti while Surdas (1483-1563), Mira Bai
(1503-73) and many others popularised. Krishna bhakti.
o Guru Nanak, the first Sikh Guru and founder of the Sikhism, was a Nirguna Bhakti Saint and social
reformer in the Punjab region. The hymns composed by him were incorporated in the Adi Granth by
the fifth Sikh Guru Arjan in 1604. Hence, pair 3 is correctly matched.
• Bhakti Movement in Bengal
o The Vaishnava bhakti was transmitted by various bhakti poet beginning with Jayadeva in the 12th
century. Jayadeva's Gita Govinda was composed in Sanskrit.
o Chaitanya (1486-1533) was another prominent Vaishnava saint of Bengal. He popularized Krishna-
bhakti in many parts of Eastern India.
o Chandidas was the first Bhakti poet-saint Bengal region in the fourteenth century. He wrote poems
based on the Krishna-Radha theme. Hence, pair 1 is not correctly matched.
• Bhakti Movement in Maharashtra
o Jnaneswar (1275-1296) was the pioneer bhakti saint of Maharashtra. He wrote an extensive
commentary on the Bhagavad Gita popularly called Jnanesvari. He was the author of many hymns
called abhangs. Hence, pair 2 is not correctly matched.
o Namdev (1270-1350) is considered to be the link between the Maharashtrian bhakti movement and
North Indian monotheistic movement. He is considered to be a part of the Varkari tradition.
o Other prominent bhakti saints of Maharashtra were Eknath (1533-99) and Tukaram (1598- 1650).
• Bhakti Movements in Other Regions
o Saiva bhakti flourished in Kashmir in the 14th century. Most prominent of the Shaiva bhakti saints
was a woman saint Lal Ded.
o Narasimha Mehta led the movement in Gujarat.
o In Assam, Sankaradeva (1449-1568) introduced bhakti both in the Brahmputra valley as well as in
Cooch-Behar. His sect is called Mahapurashiya dharma.

Q 47.D
• 'Religious paintings with a royal heritage' is the best definition for Thanjavur paintings, now better known
as Tanjore paintings. Tanjore painting ranks among the greatest traditional art forms for which India is
noted worldwide. Their themes are fundamentally mythological. These religious paintings demonstrate
that spirituality is the essence of creative work. Few art forms match the beauty and grace of Tanjore
paintings.
• Originating in Tanjavur about 300 kms from Chennai, this form of art developed at the height of
cultural evolvement achieved during the rule of the mighty Chola empire. The art form evolved and
flourished under the patronage of successive rulers. These magnificent paintings adorned the royal
dwellings and later found their way into every household. Hence statement 1 is correct.
• An extraordinary visual amalgamation of both art and craft, Tanjore paintings mainly consist of themes
on Hindu gods and goddesses, with figures of Lord Krishna in various poses and depicting various
stages of his life being the favourite. Hence statement 2 is correct.
• The characteristics of the Tanjore paintings are their brilliant colour schemes, decorative jewellery with
stones and cut glasses and remarkable gold leaf work. The liberal use of gold leaf and precious and
semi-precious stones presents a splendid visual treat. These give life to the pictures such that the
pictures come alive in a unique way. Hence statement 3 is correct.
15 www.visionias.in ©Vision IAS

Google it:- https://upscpdf.com


https://t.me/UPSC_PDF Download From > https://upscpdf.com https://t.me/UPSC_PDF

• Adorned with rubies, diamonds and other precious gemstones, and trimmed with gold foil, Tanjore
paintings were true treasures. Nowadays, however, semi-precious stones are used in place of real ones, but
the use of gold foil has not altered. The shine and glean on the gold leaves used by the Tanjore style
paintings last forever.
Q 48.B
• Recently Defence Minister has launched Konkurs Missile Test Equipment and Konkurs Launcher
Test Equipment, which have been indigenously designed and developed by Bharat Dynamics
Limited (BDL).
• Earlier, these products were imported from Russia. The two products have been launched as part of the
‘Atmanirbhar Bharat’ week.
• The Konkurs Missile Test Equipment is designed for checking the serviceability of Konkurs-M anti-tank
guided missiles. Konkurs Launcher Test Equipment is designed and developed for checking the
serviceability of Konkurs-M missile launchers. In 2019, the Indian Army (IA) has acquired a large
batch of Russian-originated 9M113M Konkurs-M ‘AT-5b Spandrel-B’ anti-tank guided missiles
(ATGMs). Hence option (b) is the correct answer.
• The Konkurs-M anti-tank guided missile (ATGM) system is designed to engage modern tanks fitted with
ERA, light armored vehicles, fortifications and engineer structures at ranges of 75 to 4,000 meters, day or
night.

Q 49.B
• The UN Convention on the Rights of Persons with Disabilities, which was adopted in 2007 as the first
major instrument of human rights in the 21st century, defines persons with disabilities as those “who have
long-term physical, mental, intellectual or sensory impairments which in interaction with various barriers
may hinder their full and effective participation in society on an equal basis with others”.
• The United Nations has released its first-ever guidelines on access to social justice for people with
disabilities to make it easier for them to access justice systems around the world. The guidelines outline a
set of 10 principles and detail the steps for implementation. The 10 principles are:
o Principle 1 All persons with disabilities have the legal capacity and, therefore, no one shall be
denied access to justice on the basis of disability.
o Principle 2 Facilities and services must be universally accessible to ensure equal access to justice
without discrimination of persons with disabilities.
o Principle 3 Persons with disabilities, including children with disabilities, have the right to appropriate
procedural accommodations.
o Principle 4 Persons with disabilities have the right to access legal notices and information in a timely
and accessible manner on an equal basis with others.
o Principle 5 Persons with disabilities are entitled to all substantive and procedural safeguards
recognized in international law on an equal basis with others, and States must provide the necessary
accommodations to guarantee due process.
o Principle 6 Persons with disabilities have the right to free or affordable legal assistance.
o Principle 7 Persons with disabilities have the right to participate in the administration of justice on an
equal basis with others.
o Principle 8 Persons with disabilities have the rights to report complaints and initiate legal
proceedings concerning human rights violations and crimes, have their complaints investigated
and be afforded effective remedies.
o Principle 9 Effective and robust monitoring mechanisms play a critical role in supporting access to
justice for persons with disabilities.
o Principle 10 All those working in the justice system must be provided with awareness-raising and
training programmes addressing the rights of persons with disabilities, in particular in the context of
access to justice. Hence option (b) is the correct answer.
Q 50.D
• The story of Malik Ambar, an African slave turned warrior, is an unusual one. Sold and bought
several times by slave dealers during his youth, fate brought him miles away from his home in Ethiopia to
India, not only did Ambar get his freedom back, but he also rose up the social ladder, got an army, vast
estates, and founded a city that today goes by the name ‘Aurangabad’. Hence statement 1 is not correct.
• In the early 1570s, Ambar was taken to the Deccan. Here he was purchased by a certain Chengiz Khan.
Khan himself was a former slave who had risen to hold the office of Peshwa or chief minister of the
Nizam Shahi sultanate of Ahmadnagar in India. However, in Deccan society, the slaves did not have a
permanent status. Upon the death of their masters, they were usually “set free” and served as per their free
will in service of powerful commanders in the Empire.
16 www.visionias.in ©Vision IAS

Google it:- https://upscpdf.com


https://t.me/UPSC_PDF Download From > https://upscpdf.com https://t.me/UPSC_PDF

• Five years after taking him on, Ambar’s master and patron Chenghiz Khan died, and Ambar was set free.
For the next 20 years, he served as a mercenary for the Sultan of neighboring Bijapur. It is here he was
given charge of a small troop and bestowed with the title “Malik”.
• In 1595, Malik Ambar returned to the Ahmadnagar Sultanate and served under another Habshi lord. This
was the time when Mughal Emperor Akbar laid eyes on the Deccan and began a significant military
expedition towards Ahmednagar. This was also Akbar’s last expedition before he passed away.
• It was really during the Mughal invasion of Ahmednagar in the late 1590s that Malik Ambar truly came
into his own. The Mughals took the capital of the Ahmadnagar Sultanate, but the wider country around it
was still in rebellion. Edging out a rival, Ambar became the leader of the resistance. Other Deccan
princes sent money and resources to Ambar to prevent inroads by Akbar and Jahangir’s armies
into their territories. Hence statement 2 is not correct.
• Apart from being an able fighter, Ambar was also a fine administrator. In 1610, after briefly expelling the
Mughals from Ahmednagar, Ambar established a new capital, a city named Khirki (present-day
Aurangabad in Maharashtra) for the sultanate.

Q 51.D
• Thumri and Thappa are popular types of Hindustani music. Hence, statement 1 is not correct.
• Thumri:
o It originated in Eastern Uttar Pradesh, mainly in Lucknow & Benares, around the 18th century.
o It is very lyrical in its structure and presentation. Hence, statement 2 is not correct.
o A romantic & erotic style of singing; also called “the lyric of Indian classical music”. Compositions
are mostly on love, separation, and devotion.
o Distinct feature: Erotic subject matter portrayed picturesquely from the various episodes of the lives
of Lord Krishna & Radha. Lyrics are typically in Brij Bhasha and are usually romantic &
religious.
o A Thumri is usually performed as the last item of a Khayal concert.
o Three main gharanas of thumri — Benaras, Lucknow and Patiala.
o Begum Akhtar is one of the most popular singers of the thumri style. Rasoolan Devi and Siddeshwari
Devi are other prominent singers.
• Tappa:
o In this style, the rhythm plays a very important role as the compositions are based on fast, subtle and
knotty constructions.
o Developed in the late 18th Century AD from the folk songs of camel riders of North-West India.
o Great use of very quick turn of phrases.
o Some of the expounders of this style: Mian Sodi, Pandit Laxman Rao of Gwalior and Shanno
Khurana.

Q 52.C
• The history of the system of music that prevailed in India from ancient times goes back to the Vedas. The
Indian musical system shows to what heights the genius of man could soar in quest of new forms of
musical expression. Apart from its entertainment value, music was cherished and practised for its quality
of lifting mankind to a nobler plane, enabling the soul to attain eternal bliss.
• The earliest treatise we have on music is the Natya Sastra of Bharata. Other treatises on music after
Bharata, such as the Brihaddesi of Matanga, Sangeeta Ratnakara of Sharangadeva, Sangeet Sudhakara
of Haripala, Swaramelakalanidhi of Ramamatya, etc., provide us a fund of information about the
different aspects of music and its development during the different periods.
• The ancient Tamils of South India had also developed a highly evolved system of music with its solfa
methods, concordant and discordant notes, scales and modes, etc. A number of instruments were also used
to accompany song and dance. The Tamil classic of the 2nd century A.D. titled the Silappadhikaram
contains a vivid description of the music of that period. The Tolkappiyam, Kalladam and the contributions
of the Saivite and Vaishnavite saints of the 7th and 8th centuries A.D. also serve as resource material for
studying musical history.
• The course of the evolution of Indian music saw the emergence of two different sub systems as
Hindustani and Carnatic music. In Haripala's "Sangeeta Sudhakara", written in the 14th century A.D., the
terms Carnatic and Hindustani are found for the first time. The two distinct styles, Hindustani and
Carnatic came into vogue after the advent of the Muslims, particularly during the reign of the Mughal
Emperors of Delhi.

17 www.visionias.in ©Vision IAS

Google it:- https://upscpdf.com


https://t.me/UPSC_PDF Download From > https://upscpdf.com https://t.me/UPSC_PDF

• The advent of Purandaradasa, in the year 1484, marked a very important landmark in the development of
Carnatic music. He effected such a thorough systematization and refinement in the art, that, up to the
present day, it has remained the same.
• Moving into the 17th century, the history of Carnatic music saw the epoch-making scheme of the 72
Melakartas, introduced by Venkatamakhi and laid down in his monumental work Chaturdandi
Prakasika in the year 1620 A.D.
• The Melakarta scheme is a highly comprehensive and systematic formula which includes within its fold
all the modes used in ancient as well as modern systems of the music of the different parts of the world.
The scheme opened out new vistas of raga creation and later composers like Tyagaraja invented many a
beautiful raga by following it. Hence option (c) is correct answer.

Q 53.C
• Statement 1 is correct: ESIC is implementing the Atal Bimit Vyakti Kalyna Yojna under which
unemployment benefit is paid to the workers covered under the ESI Scheme. The ESI Corporation has
decided to extend the scheme for one more year upto 30th June 2021. It has been decided to relax the
existing conditions and the amount of relief for workers who have lost employment during the
Covid-19 pandemic period.
• Statement 2 is correct: The Atal Bimit Vyakti Kalyan Yojana is being implemented by Employee
State Insurance Corporation. The scheme was approved under the Employees State Insurance Act,
1948. The scheme aims to support the people who have lost their jobs due to changing employment
patterns due to an international crisis or a domestic crisis. Currently, the change in the employment pattern
is due to COVID-19. The scheme was introduced in 2018. The scheme provides 25% of average per day
earning in the previous four contribution periods.
• The eligibility criteria for availing the relief has also been relaxed, as under:
o The payment of relief has been enhanced to 50% of the average wages from earlier 25% of average
wages payable upto a maximum of 90 days of unemployment.
o Instead of the relief becoming payable 90 days after unemployment, it shall become due for payment
after 30 days.
o The Insured Person can submit the claim directly to the ESIC Branch Office instead of the claim
being forwarded by the last employer and the payment shall be made directly in the bank account of
IP.
o The Insured Person should have been insurable employment for a minimum period of 2 years before
his/her unemployment and should have contributed for not less than 78 days in the contribution period
immediately preceding to unemployment and minimum 78 days in one of the remaining 3
contribution periods in 02 years prior to unemployment.

Q 54.D
• During the 9th century, in the Ganga Valley, Kannauj became the centre of gravity due to its strategic
and geographical potentiality. The control over Kannauj implied control over the eastern and western
parts of the Ganga doab which was very fertile. It was also interconnected with land and water routes.
Therefore the three leading contemporary powers such as the Palas, the Pratiharas and the Rashtrakutas
clashed over the possession of Kannauj.
• The Palas were primarily centered in Eastern India, the Pratiharas in Western India and the
Rashtrakutas in the Deccan. But all the three power tried to control the Ganga plains, especially
Kannauj. This is known as the tripartite struggle.

Q 55.B
• The Magadha Empire came into existence when the four Mahajanpada- Magadha, Kosala, Vatsa and
Avanti were engaged in the struggle for supremacy from the 6th Century BC to 4th Century BC.
Ultimately, Magadha emerged as the most powerful and prosperous kingdom in North India.
• Magadha rose into prominence under the rule of Bimbisara, who was a contemporary of Buddha.
o He was the first king to have a standing army and contemporary to the Buddha.
o He was the first who started matrimonial alliances to strengthen its position as a king after defeating
Brahamadutta
• Magadha's most serious rival was Avanti with its capital at Ujjain. It was then ruled by Chanda
Pradyota Mahasena.
• Bimbisara and Pradyota arrived at a framework of friendship for mutual benefit and later, Bimbisara
even sent his court physician Jivaka to cure Pradyota of jaundice. This friendship did not last during
their successors.
18 www.visionias.in ©Vision IAS

Google it:- https://upscpdf.com


https://t.me/UPSC_PDF Download From > https://upscpdf.com https://t.me/UPSC_PDF

• The earliest capital of Magadha was at Rajgir, which was called as Girivraja, for it was surrounded by
five hills.
• Bimbisara was killed by his son Ajatasatru.
o He was a contemporary of both Mahavira and Gautama Buddha.
o He was also instrumental in convening the First Buddhist Council at Rajagriha soon after the
death of Buddha.
• Ajatasatru was succeeded by Udayin, whose reign was famous for building Pataliputra and making it
the capital of Magadha.
• Sisunagas succeeded udayin, the last known ruler of the Haryanka dynasty.
o Their greatest achievement was the destruction of the power of Avanti, which brought to an
end, the 100-year-old rivalry between Magadha and Avanti. Hence, option (b) is the correct
answer.
• Magadha was later ruled by Nandas, then the mighty Mauryas, Sungas, Kanvas, etc.

Q 56.D
• Both statement 1 and statement 2 are not correct: Recently Commerce and Industry Minister launched
the National GIS-enabled Land Bank system. The system is being developed by the integration of the
Industrial Information System (IIS) with State GIS Systems. The project was launched in 6 states. It
enables investors to get real-time information about the availability of industrial land and resources.
• IIS portal is a GIS-enabled database of industrial areas/clusters across the states. More than 3,300
industrial parks across 31 States/UTs covering about 475,000 hectares of land have been mapped on the
System. The information available includes forest, drainage; raw material heat maps (agricultural,
horticulture, mineral layers); multiple layers of connectivity. National GIS-enabled Land. The IIS adopts a
committed approach towards resource optimization, industrial up-gradation and sustainability. The
initiative is being supported by Invest India, National Center of Geo-Informatics (NCoG), National e-
Governance Division (NeGD), Ministry of Electronics & Information Technology and Bhaskaracharya
Institute for Space Applications and Geo-Informatics (BISAG).
Q 57.B
• Statement 2 is correct: Recently the National Commission on Population submitted its report on
population projections for the period of 2011-36. The commission was set up by the Ministry of
Health and Family Welfare. India’s population is expected to grow by 25%, with reference to 2011, to
1.52 billion by 2036, according to the final report of the technical group on population projections dated
July 2020. The group was constituted by the National Commission on Population (NCP) under the
Ministry of Health and Family Welfare with the mandate to provide population projections for the period
2011 to 2036.
• Statement 1 is not correct: The National Commission on Population was constituted in May 2000
to review, monitor and give directions for the implementation of the National Population Policy (NPP),
2000 with a view to meeting the goals set out in the Policy, to promote inter-sectoral coordination, involve
the civil society in planning and implementation, facilitate initiatives to improve performance in the
demographically weaker States in the country and to explore the possibilities of international cooperation
in support of the goals set out in the National Population Policy. It is not constituted every year.
Q 58.D
• Gandhara School was based on Greco-Roman norms encapsulating foreign techniques and an alien spirit.
It is also known as the Graeco-Buddhist School of art.
• It mostly flourished in the areas of Afghanistan and present North-Western India. The prominent locations
were Taxila, Peshawar, Begram, and Bamiyan.
• The Gandhara school of Art reached its peak under the reign of Kanishka the Great from 127 to 151 AD.
This school inherited Greco-Buddhist art. This school for the first time depicted Buddha as a human
form. Hence statement 1 is correct.
• Gandharan sculptures show strong Greek influences in the depiction of a ‘man-god’ and of wavy hair,
sandals and extensive drapery. The depiction of Buddha as a ‘man-god’ in Gandharan sculpture is
believed to be inspired by Greek mythology. Some examples of Gandharan art depict both Buddha and
the Greek god, Hercules.
• Stucco plaster, which was commonly observed in Greek art, was widely used in Gandharan artwork for
the decoration of monastic and cult buildings. Grey sandstone (Blue-grey Mica schist to be precise) is
used in Gandhara School of Art. Hence statement 2 is correct.
• The expression of calmness is the centre point of attraction of Gandhar Buddha. Overall the image
is very expressive and calmness remains the centre point of attraction in the Gandhara style of
Buddha. Hence statement 3 is correct.
19 www.visionias.in ©Vision IAS

Google it:- https://upscpdf.com


https://t.me/UPSC_PDF Download From > https://upscpdf.com https://t.me/UPSC_PDF

Q 59.B
Nyaya (Gautama Muni):
• Nyaya Philosophy states that nothing is acceptable unless it is in accordance with reason and
experience (scientific approach). Nyaya is considered as a technique of logical thinking.
• Nyaya Sutras say that there are four means of attaining valid knowledge: perception, inference,
comparison, and verbal testimony. Hence option (b) is correct.
Vedanta:
• The Vedanta, or Uttara Mimamsa, the school concentrates on the philosophical teachings of the
Upanishads (mystic or spiritual contemplations within the Vedas), rather than the Brahmanas (instructions
for ritual and sacrifice).
Vaisheshika (Kanada):
• The basis of the school's philosophy is that all objects in the physical universe are reducible to a finite
number of atoms and Brahman is regarded as the fundamental force that causes consciousness in these
atoms.
Samkhya (Kapila):
• Samkhya is the oldest of the orthodox philosophical systems, and it postulates that everything in reality
stems from purusha (self, soul or mind) and prakriti (matter, creative agency, energy). Purush cannot be
modified or changed while Prakriti brings change in all objects.

Q 60.A
• Recently, Aryabhatta Research Institute of Observational Sciences (ARIES), Nainital an
autonomous research institute under the Department of Science and Technology (DST), Govt. of
India and Hemvati Nandan Bahuguna Garhwal University, (HNBGU) (a central University) Srinagar,
Pauri Garhwal, Uttarakhand, jointly organized the three-day international conference on ‘Aerosol Air
Quality, Climate Change and Impact on Water Resources and Livelihoods in the Greater
Himalayas’.
• ARIES, Nainital, started research activities in the field of atmospheric science and climate change about
two decades ago. It has installed a battery of modern instruments at Manora Peak, a strategically
important location in the Central Himalayan region, for in situ measurements of various optical, physical
and chemical properties of aerosols and trace gases. The ARIES has also established both national and
international collaborations in these areas.

Q 61.B
• Indian dance is a blend of nritta (the rhythmic elements), nritya (the combination of rhythm with
expression) and natya (the dramatic element).
o Nritta is the rhythmic movement of the body in dance. It does not express any emotion.
o Nritya is usually expressed through the eyes, hands and facial movements. Nritya combined with
nritta makes up the usual dance programs. Nritya comprises abhinaya, depicting rasa (sentimental)
and Bhava (mood).
o Natya corresponds to drama. It means dramatic representation or drama with speech, music and
dancing.
• The ideal dance postures of the body are depicted in the Shilpa Shastra and there are 4 types of Bhangas
(postures), the deviations of the body from the central erect position. These four Bhangas are Abhanga,
Samabhanga, Atibhanga and Tribhanga.
o Abhanga signifies "off-center", an iconographic term for a slightly askew standing position.
o Samabhanga is the equal distribution of the body limbs on a central line, whether standing or sitting.
o Atibhanga is the great bend with the torso diagonally inclined and the knees bent.
o Tribhanga is the triple bend with one hip raised, the torso curved to the opposite side and the head
tilted at an angle.

Q 62.D
• The major religious movements were brought about by the mystics. They contributed to religious ideas
and beliefs. Bhakti saints like Vallabhacharya, Ramanuja, Nimbaraka brought about new philosophical
thinking which had its origin in Shankaracharya’s Advaita (non-dualism) philosophy.
• Vishistadvaita of Ramanujacharya
20 www.visionias.in ©Vision IAS

Google it:- https://upscpdf.com


https://t.me/UPSC_PDF Download From > https://upscpdf.com https://t.me/UPSC_PDF

o Vïshistadvaita means modified monism. The ultimate reality according to this philosophy is Brahman
(God) and matter and soul are his qualities.
• Dvaita of Madhavacharya
o The literal meaning of dvaita is the dualism which stands in opposition to non-dualism and monism of
Shankaracharya. He believed that the world is not an illusion (Maya) but a reality full of differences.
• Dvaitadvaita of Nimbaraka.
o Dvaitadvaita means dualistic monism. According to this philosophy, God transformed himself into a
world and soul. This world and soul are different from God (Brahman). They could survive with the
support of God only. They are separate but dependent.
• Suddhadvaita of Vallabhacharya
o Vallabhacharya wrote commentaries on Vedanta Sutra and Bhagavad Gita. For him. Brahman (God)
was Sri Krishna who manifested himself as souls and matter. God and soul are not distinct, but one.
The stress was on pure non-dualism. His philosophy came to be known as Pushtimarga (the path of
grace) and the school was called Rudrasampradaya.
• Hence, pairs 1 and 3 are correctly matched.

Q 63.B
• Thirukkural (Tamil: “Sacred Couplets”), also called Kural, is the most celebrated of the Patiren-
kirkkanakku (“Eighteen Ethical Works”) in Tamil literature and work that has had an immense influence
on Tamil culture and life. It is attributed to the poet Tiruvalluvar who lived in Tamil Nadu in the 6th
century, though some scholars assign an earlier date (1st-century bc). In its practical concerns, aphoristic
insights into daily life, and universal and timeless approach, the Tirukkural has been compared to the great
books of the world’s major religions.
• Statement 1 is not correct: Thirukkural is concerned with an ethical lifestyle and is very practical in its
approach and focus materials. It has as its foremost moral imperatives the avoidance of killing and the
avoidance of falsehood. It also commends to the reader a feeling of compassion for all individuals,
regardless of caste or creed. While on the other hand, the Sangam literature is almost entirely religious.
The poems are concerned with two main topics: those of the first five collections are on love (akam), and
those of the next two are on heroism (puram), including the praise of kings and their deeds. Paripatal, the
eighth collection, contains poems of both types. Many of the poems, especially on heroism, display great
freshness and vigour and are singularly free from the literary conceits of much of the other early and
medieval literature of India. Thirukkural is widely regarded as a secular work which despite some
references to God, does not venture to propound the tenets of any particular religion or godhead.
• Statement 2 is correct: Its 133 sections of 10 couplets each are divided into three books: aram (virtue),
porul (government and society), and kamam (love). The first section opens with praise of God, rain,
renunciation, and a life of virtue. It then presents a world-affirming vision, the wisdom of human
sympathy that expands from one’s family and friends to one’s clan, village, and country.

Q 64.B
• The Mauryan king headed the judicial system. The Arthashastra mentions courts to try law-breakers and
settle disputes at various levels. The king held his own court to provide justice and, according to some
sources, the chief justice had four or five judges to assist him.
• At the local level, there were courts formed by citizens, apart from courts formed by trade guilds and
village assemblies.
• There were civil and criminal courts which are mentioned in the Arthashastra. The dharmasthiya were
the civil court and the kantakasodhana was the criminal wing. The former dealt with civil matters and
was presided over by three amatyas and three dharmasthas.
• There were rules for court procedures and the manner in which an agreement could be nullified. The
criminal courts were special tribunals presided over by three amatyas or pradestris aided by spies and
informers. The source of law for Mauryas was Sukraneeti written in post- Vedic age.
• Hence option (b) is the correct answer.

Q 65.D
• Nalanda University is located in Rajgir, near Nalanda, Bihar, India. It stands for a university that attracted
students and scholars from across Asia and even farther away. It was developed under the patronage of
the Kumaragupta of the Gupta dynasty and was the first international residential university in the
world. Hence statement 1 is not correct.
• The top ten eminent rulers of the Gupta empire in India are:
o Chandra Gupta I (320-335 or 340 A.D.)
21 www.visionias.in ©Vision IAS

Google it:- https://upscpdf.com


https://t.me/UPSC_PDF Download From > https://upscpdf.com https://t.me/UPSC_PDF

Google it:- https://upscpdf.com


https://t.me/UPSC_PDF Download From > https://upscpdf.com https://t.me/UPSC_PDF

Google it:- https://upscpdf.com


https://t.me/UPSC_PDF Download From > https://upscpdf.com https://t.me/UPSC_PDF

Q 72.C
• The ownership of large tracts of land came to be regarded as the prized possession for chieftains, as
increased land brought more agricultural surplus and higher tribute collection to a chieftain’s Janapada or
territory. It was this search for individual ownership of more and more land that finally culminated into
the emergence of a large kingdom or Mahajanapadas around and 6th century BC.
o Mahajanapadas: capital
o Anga Champa
o Magadha. Girivraja| Rajagriha
o Vatsa Kausambi Allahabad
o Avanti. Ujjain
o Vajji. Vaishali
Hence option (c) is the correct answer.

Q 73.D
• One of the oldest surviving mural painting of the Indian subcontinent, Ajanta caves were carved between
the 2nd century BC to 5th Century AD out of volcanic rocks. It consists of 29 caves, carved in a shoe
horse shape.
• These Buddhist caves are quite popular for their exquisite mural paintings. Murals in cave no. 9 and 10
belong to the Shunga period, while the rest belongs to the Gupta Period.
• Some important themes of Ajanta are taken from the jataka stories of Buddha’s former lives as
Bodhisattva, the life of Gautam Buddha, etc. For example, Painting of Bodhisattva in tribhanga pose in
cave no 1: Vajrapani, Manjusri, and Padmapani (Avalokitesvara).
• The other important paintings of Ajanta are Dying princess in cave no 16, Shibli Jataka and the
scenes of Maitri poshaka Jataka and Mahajanaka Jataka are well depicted.
• Hence option (d) is the correct answer.

Q 74.C
• Fresco Painting: Technically, if the painting is created (or we can say that the colours are applied) on a
plaster which is still wet, the painting is called fresco (Italian word which means ‘fresh’). While painting
a fresco, no binder is required to be added to the mineral pigments that are applied because the
chemical reaction of the wet plaster with the carbon dioxide in the air creates a hard layer of calcium
carbonate and the pigments are securely fixed in the calcium carbonate layer while drying.
• Tempera paintings:
• Tempera paintings require a binder to be added to the pigment colours to make them stick to the
plaster. They are also quite stable but susceptible to damage by moisture and dampness.
• Tempera paintings are done on dry plaster, so there is no stress of the time deadline. Hence statement
1 is correct.
• In India Paintings were executed in the traditional tempera technique. The frescoes on the ceilings
and walls of the Ajanta caves and Bagh caves were painted between 200 B.C. and 600 A.D. and are
the oldest known frescoes in India. Hence statement 2 is correct.

Q 75.B
• Kavirajamarga is the earliest available Jain work on rhetoric, poetics and grammar in the Kannada
language. It was inspired by or written in part by the famous Rashtrakuta King “Nrupathunga”
Amoghavarsha. Hence statement 1 is not correct.
• Bhadrabahu was the last acharya of the undivided Jain sangha. He was the spiritual teacher of
Chandragupta Maurya, the founder of the Mauryan Empire. He converted Chandragupta Maurya to
Jainism. Hence statement 2 is correct.
• Gangs, Kadambs, Chalukyas, Rastrakutas had patronized Jainism. Rastrakutas was the last great
patrons of Jainism. Hence statement 3 is correct.

Q 76.B
• Yoga literally means the union of two principal entities. Yogic techniques control body, mind & sense
organs, thus considered as a means of achieving freedom or mukti. This philosophy admits the existence
of God as a teacher and guide hence consider God as a means not as an end to achieve salvation. Hence
statement 1 is not correct and 3 is correct.
• Identified in the Yoga Sutras of Patanjali, the five Kleshas or ‘afflictions’ are Avidya (ignorance), Asmita
(egoism), Raga (attachment), Dvesa (aversion or hatred), and Abinivesah (clinging to life and fear of
death). Some of the kleshas afflict us on a daily basis at a subtle level, yet others can be overwhelming
24 www.visionias.in ©Vision IAS

Google it:- https://upscpdf.com


https://t.me/UPSC_PDF Download From > https://upscpdf.com https://t.me/UPSC_PDF

and cause great pain. Overcoming the kleshas is thought to lead to the end of suffering, and the ultimate
liberation from the repeated cycle of birth, death and rebirth, known as the wheel of Samsara.
• Yoga philosophy has described the path to getting rid of these kleshas by attaining the highest stage
of samadhi through Ashtanga yoga (Yama-Niyama-Asana-Pranayama-Pratyahara-Dharan-Dhyan
and Samadhi). Hence statement 2 is correct.

Q 77.C
• As per Abhinaya Darpan (5th–4th century BC), Nandikeshwara’s famous treatise on dance, an act has
been broken into three basic elements:
o Nritta – It refers to the basic dance steps, performed rhythmically but devoid of any expression or
mood. Hence statement 1 is not correct.
o Natya - It means dramatic representations and refers to the story that is elaborated through the
dance recital. Hence statement 2 is correct.
o Nritya - Nritya refers to the sentiment and the emotions evoked through dance. It includes the
mime and the different methods of expression including mudras in the dance. Hence statement 3 is
not correct.
• Nandikeshwara further elaborates the Nayaka-Nayika Bhav, in which the eternal deity is seen as the hero
or Nayaka and the devotee who performs the dance is the heroine of the act, the Nayika

Q 78.C
• Statement 1 is correct: With the aim to promote a comprehensive approach to decarbonize
transport in India, Vietnam, and China, NDC–TIA is a joint programme, supported by the
International Climate Initiative (IKI) of the German Ministry for the Environment, Nature
Conservation and Nuclear Safety (BMU) and implemented by a consortium of seven organisations,
namely:
1. Deutsche Gesellschaft für Internationale Zusammenarbeit (GIZ) GmbH
2. International Council on Clean Transportation (ICCT)
3. World Resources Institute (WRI)
4. International Transport Forum (ITF)
5. Agora Verkehrswende (AGORA)
6. Partnership on Sustainable, Low Carbon Transport (SLoCaT) Foundation
7. Renewable Energy Policy Network for the 21st Century e.V. (REN21)
• Statement 2 is correct: The India Component is implemented by six consortium organisations, all except
SLoCaT. On behalf of the Government of India, NITI Aayog, the country’s premier policy think
tank, will be the implementing partner. The NDC-TIA programme has a duration of 4 years and will
allow India and other partner countries to achieve accountable long-term targets by making a sectoral
contribution through

Q 79.A
• Buddha advocated avoiding the two extremes of indulgence in worldly pleasure and the practice of
strict abstinence and asceticism. He ascribed instead of the 'Madhyam Marg' or the middle path
which was to be followed. Hence statement 2 is not correct.
• According to him, everyone was responsible for their own happiness in life, stressing upon the
individualistic component of Buddhism.
• The main teachings of Buddhism are encapsulated in the basic concept of four noble truths or ariya-
sachchani and eightfold path or astangika marg.
• The essence of Buddhism is the attainment of enlightenment. It points to a way of life that avoids self-
indulgence and self-denial. There is no supreme god or deity in Buddhism.
• The ultimate goal of Buddha’s teaching was the attainment of nirvana which was not a place but
experience and could be attained in this life. Hence statement 3 is correct.
• Buddhism believes in the transmigration of souls. Hence statement 1 is correct.
Q 80.D
• In the world context, the New Stone or the Neolithic age began in 9000 BC. The settlement lay on the
edge of the Indus plains. It is called one of the largest Neolithic settlements between the Indus and the
Mediterranean. Though the earliest settlers raised domestic animals and produced cereals, they were
disturbed around 5500 BC by floods. Agricultural and other activities were resumed around 5000 BC with
the help of both stone and bone tools. The Neolithic people of this area produced wheat and barley
from the outset. They domesticated cattle, sheep, and goats in the initial stage. Hence options 1, 2
and 3 are correct.
25 www.visionias.in ©Vision IAS

Google it:- https://upscpdf.com


https://t.me/UPSC_PDF Download From > https://upscpdf.com https://t.me/UPSC_PDF

• The people of the Neolithic age used tools and implements of polished stone. Hence option 4 is
correct.
• At the level of Neolithic, extensive knowledge of metallurgy is not indicated. Limited use of metal had
been seen in the Chalcolithic period. Hence option 5 is not correct.

Q 81.D
• The city of Dholavira was located on Khadir Beyt in the Rann of Kutch, where there were fresh water and
fertile soil. Unlike some of the other Harappan cities, which were divided into two parts, Dholavira was
divided into three parts, and each part was surrounded with massive stone walls, with entrances through
gateways. There was also a large open area in the settlement, where public ceremonies could be held.
Other finds include large letters of the Harappan script that were carved out of white stone and perhaps
inlaid in wood. This is a unique find as generally Harappan writing has been found on small objects such
as seals. Hence statement 2 is correct.
• Chanhudaro is an archaeological site belonging to the Indus Valley Civilization. The site is located 130
kilometres south of Mohenjodaro, in Sindh, Pakistan. The settlement was inhabited between 4000 and
1700 BCE and is considered to have been a centre for manufacturing carnelian beads. This site is a group
of three low mounds that excavations have shown were parts of a single settlement, approximately 5
hectares in size. Chanhudaro was the only city without a citadel in the Indus Valley
civilization. Hence statement 1 is correct.
• Rakhigarhi is the site of a pre-Indus Valley Civilisation settlement going back to about 6500 BCE. Later,
it was also part of the mature Indus Valley Civilisation, dating to 2600-1900 BCE. The site is located in
the Saraswati/Ghaggar-Hakra River plain, some 27 km from the seasonal Ghaggar river. Rakhigarhi is
the largest Indian site of IVC. Hence statement 3 is correct.

Q 82.B
• Although artisans, peasants, priests and warriors appear even in the earlier portions of Rigveda, the
society of vedic period as a whole was tribal, pastoral, semi-nomadic and egalitarian.
• Spoils of war and cattle constituted the main forms of property.
• Cattle and women slaves were generally given as gifts. Gifts of cereals were hardly mentioned because
they were not produced at any considerable scale. Hence, statement 4 is not correct.
• Therefore apart from the war booty captured in war, there was no other substantial revenue source
for maintenance of princes and priests. Hence, statement 3 is correct.
• It was possible to have high ranks, but not high social classes
• Princes and priests employed women slaves for domestic service, but their number may not have been
large. Hence, statement 1 is correct.
• The Rigvedic society did not have a serving order in the form of the Sudras. Hence, statement 2 is
correct.

Q 83.A
Daimabad:
• It is a deserted village and an archaeological site on the left bank of the Pravara River, a tributary of
the Godavari River in Shrirampur taluka in Ahmednagar district of Maharashtra state in
India. This site was discovered by B. P. Bopardikar in 1958. (Hence option 1 is correct.)
Kot Diji:
• It is an archaeological site located near an ancient flood channel of the Indus River in Pakistan, 15 miles
(25 km) south of the city of Khairpur in Sindh province. (Hence option 2 is not correct).
Suktagendor:
• It was located around 55 km from the shore of the Arabian Sea on the Bank of Dasht River near the
Iran Border. (Hence option 3 is not correct)
• It was an important coastal town. Suktagendor is considered to be the western border of the Indus Valley
Civilization
Surkotada:
• It is an archaeological site located in Rapar Taluka of Kutch district, Gujarat, India which belongs to the
Indus Valley Civilisation (IVC). (Hence option 4 is correct.)

26 www.visionias.in ©Vision IAS

Google it:- https://upscpdf.com


https://t.me/UPSC_PDF Download From > https://upscpdf.com https://t.me/UPSC_PDF

Q 84.C
• Dhrupad denotes both verse form of the poetry and the style in which it is sung. Although Dhrupad
consolidated its position as a classical form of music in the 13th century, it reached its zenith in the
court of emperor Akbar. Akbar employed and patronised musical masters like Baba Gopal Das, Swami
Haridas and Tansen, who was considered to be one of the Navaratna or nine gems of the Mughal
court. Hence, statement 1 is correct.
• The word ‘Khyal’ is derived from Persian and means “idea or imagination”. Khyal is based on the
repertoire of short songs ranging from two to eight lines. The origin of this style was attributed to Amir
Khusrau and not Tansen. Hence, statement 2 is not correct
• Comparable to the vanis of the dhrupads, gharanas are part of the khyal music. Hence, statement 3 is
correct.

Q 85.D
• Akbar is known for his liberal ideas and liberal religious policy. He adopted a policy of mutual
understanding and reconciliation among followers of different faiths and equality of all religions. He tried
to harmonize the relations.
• One of the first actions which Akbar took after he had taken power in his own hands, was to abolish the
poll tax or jizyah which the non-Muslims were required to pay in a Muslim state. Although it was not a
heavy tax, it was disliked because it made a distinction between subject and subject.
• At the same time, Akbar abolished the pilgrim tax on bathing at holy places such as Prayag,
Banaras, etc. He also abolished the practice of forcibly converting prisoners of war to Islam. This
laid the essential foundation of an empire based on equal rights to all citizens, irrespective of their
religious beliefs. Hence, statements 1 and 2 are correct.
• The liberal principles of the empire were strengthened by bringing able Hindus into the nobility.
• In 1575, Akbar built a hall called Ibadat Khana or the Hall of Prayer at his new capital, Fatehpur Sikri. To
this, he called selected theologians, mystics, and those of his courtiers and nobles who were known for
their scholarship and intellectual attainments. Akbar discussed religious and spiritual topics with
them. Akbar opened the Ibadat Khana to people of all religions— Christians, Zoroastrians, Hindus, Jains,
even atheists.
• Akbar tried to emphasize the concept of sulh-i-kul or peace and harmony among religions in other ways
as well. He set up a big translation department for translating works in Sanskrit, Arabic, Greek, etc., into
Persian. Thus, the Singhasan Battisi, the Atharva Veda, and the Bible were taken up first for translation.
These were followed by the Mahabharata, the Gita, and the Ramayana. Many others, including the
Panchatantra and works of geography, were also translated. The Quran was also translated into Persian,
perhaps for the first time.

27 www.visionias.in ©Vision IAS

Google it:- https://upscpdf.com


https://t.me/UPSC_PDF Download From > https://upscpdf.com https://t.me/UPSC_PDF

• Akbar founded a new religion known as ‘Din-i-Ilahi’ based on the common points of all
religions. Din-i Ilahi “the religion of God,” idea was to combine Islam and Hinduism into one faith,
but also to add aspects of Christianity, Zoroastrianism, and Jainism. Hence, statement 3 is correct.

Q 86.D
• Kishangarh was a very small state but a very different type of artistic development took place here
• This insignificant state was founded by Kishan Singh in the initial years of the 17th century. The
tremendous development of art took place under the local ruler Sawant Singh, a scholar, a critic and a
renowned poet and well versed in Hindi, Sanskrit, Persian; who himself was a poet and used to write with
a pen name Nagari Das.
• It is a totally different style with highly exaggerated features like long necks, large, almond-shaped eyes,
and long fingers. This style of painting essentially depicts Radha and Krishna as divine lovers, and
beautifully portrays their mystical love. Hence statement 1 is correct.
• Nagari Das took his poetic inspirations from a painter in the town called Nihal Chand, who was an
inimitable master in depicting the Radha and Krishna. Nihal Chand was the court painter of Nagri
Das. He painted the celebrated Bani Thani. Hence statements 2 and 3 are correct.
• Sawant Singh was so deeply fell in love with the imaginary lover that he abdicated the throne within a
span of 9 years to take recourse in Brindavan and worship his lover Radha as Bani Thani and himself live
like his lord Krishna.

Q 87.D
• The Mahābhasya attributed to Patanjali, is a commentary on selected rules of Sanskrit grammar from
Panini’s treatise, the Ashtadhyayi, as well as Kātyāyana's Varttika, an elaboration of Pāṇini's grammar. It
is dated to the 2nd century BCE.
• Mudrarakshasa, written by Vishakadatta (Translated by R S Pandit) in 6th century CE, is a political
thriller set in an interesting period in Indian history. It is the time of Chanakya, Chandragupta Maurya,
and the Nandas.
• The Brihatkatha or Lord Shiva's narrative to his wife Parvati is featured in Gunadhya's epic
composition 'Katha Sarita Sagara' in Sanskrit. Brihatkatha, literally meaning the big (vrihat) story (katha),
is an ancient Indian epic, and a great many classic Indian fables like the Panchatantras, Hitopadesha, etc.
• Silappadikaram was written by Ilango Adigal in the second century A.D. It is a tragic story of a
merchant, Kovalan of Puhar who falls in love with a dancer Madhavi, neglecting his own wife, Kannagi,
who in the end revenges the death of her husband at the hands of the Pandyan King and becomes a
goddess.
• Hence option (d) is the correct answer.

Q 88.A
• Recently, a Price Monitoring and Resource Unit (PMRU) has been set up in Karnataka under the aegis of
National Pharmaceutical Pricing Authority (NPPA), Department of Pharmaceuticals, Ministry of
Chemicals and Fertilizers, Government of India.
• PMRU will function at the State level under the direct supervision of the State Drug Controller for
increasing outreach of NPPA. PMRUs are societies registered under the Societies Registration Act having
its own Memorandum of Association/ Bye laws. NPPA, under its Central Sector Scheme named
Consumer Awareness, Publicity and Price Monitoring (CAPPM), has already set up of PMRUs in 12
States/ UTs.
• The primary function of PMRUs is to assist NPPA in monitoring of prices of drugs, ensuring
availability of drugs and raising consumer awareness. They act as collaborating partners of NPPA
with information gathering mechanism at the grass-roots level. They will render necessary technical
assistance to both the NPPA and the respective State Drug Controllers of States/ Union Territories. hence
option (a) is the correct answer.
• NPPA has been working in tandem with State Governments during the COVID-19 pandemic to ensure
seamless availability of life saving essential medicines including HCQ, paracetamol, vaccines, insulin and
medicines under COVID protocol. Working together with State Governments, NPPA has strived to ensure
that there is no shortage of drugs throughout the country. The PMRUs are expected to strengthen drug
security and affordability at regional levels.

28 www.visionias.in ©Vision IAS

Google it:- https://upscpdf.com


https://t.me/UPSC_PDF Download From > https://upscpdf.com https://t.me/UPSC_PDF

Q 89.B
• Gatka
o It is a weapon based martial art form, performed by the Sikhs of Punjab. The name ‘Gatka’ refers to
the one whose freedom belongs to grace. Gatka features the skillful use of weapons, including stick,
Kirpan, Talwar and Kataar. The attack and defence in this art form is determined by the various
positions of hands and feet and the nature of weapon used. It is displayed on a number of celebrations
in the State including fairs.
• Thoda
o Originating from Himachal Pradesh, Thoda is a mixture of martial arts, sports and culture. It
takes place during the Baisakhi festival in April every year. A number of community prayers are
done so as to invoke the blessings of Goddesses Mashoo and Durga, the principal deities. The
martial art relies on a player’s skill of archery. Thoda can be dated back to Mahabharata, the
times when bows and arrows were used in the epic battle, in the Valleys of Kullu and Manali.
o The game is played in a marked court so as to ensure a certain degree of discipline. The two teams
are called Pashis and Saathis, who are believed to be the descendants of Pandavas and
Kauravas of Mahabharata. The archers aim for the leg, below the knee, as there are negative points
for striking any other parts of the body. Hence, option (b) is the correct answer.
• Mardani Khel
o This is a traditional Maharashtrian armed martial art, which is practiced widely in the district of
Kolhapur. Mardani Khel focuses primarily on skills of weaponry, especially swords, swift
movements and use of low stances that is suited to its originating place, the hill ranges. It is known for
the use of the unique Indian Patta (sword) and Vita (corded lance). Famous practitioners included
Shivaji.
• Sarit Sarak
o It is an unarmed art form that uses hand-to-hand. combat. Their history can be traced back to the
17th century when it was successfully used by the Manipuri kings to fight with British. The
capture of this region by the British witnessed the ban on this art form, however post-independence
there was a resurfacing of the same.

Q 90.A
• In Puppet Theatre various forms, known as puppets, are used to illustrate the narratives. The puppeteer
narrates his story in verse or prose, while the puppets provide the visual treat. Stories adapted from
puranic literature, local myths and legends usually form the content of traditional puppet theatre in India
• Excavations at several Harappan sites have revealed a number of toys whose body parts can be
manipulated with strings.
• There are basic four kinds of puppets used in India as follows:
o String Puppets – This includes Kathputli of Rajasthan, Kundhei of Odisha, Gombeyetta of
Karnataka and Bomallattam art of Tamil Nadu.
o Shadow Puppets – This includes the Togalu Gombeyatta of Karnataka, Tholu Bommalata of
Andhra Pradesh, Ravanachhaya of Odisha. Hence, option (a) is the correct answer.
o Rod Puppets – This includes Putul Nautch of West Bengal and Yampuri of Bihar
o Glove Puppets – Important form is Pavakoothu of Kerala.

Q 91.C
• Ashoka, the greatest of the Mauryas, formulated the policy of dhamma which occupies a significant place
in the Mauryan polity. Though the word dhamma is sometimes associated with religion, actually it was a
secular policy having certain social and political implications.
• Ashoka established the idea of paternal kingship. He regarded all his subjects as his children and believed
it the king’s duty to look after the welfare of the subjects.
• Through his edicts, he said everybody should serve parents, revere teachers, and practice ahimsa and
truthfulness. He asked everyone to avoid animal slaughter and sacrifice. He expounded humane treatment
of animals, servants and prisoners. Thus the broad objective of the Dhamma was to preserve social
order. Hence statement 2 is correct.
• He advocated tolerance towards all religions. He sought conquest through Dhamma and not war. He sent
missions abroad to spread the word of the Buddha. Notably, he sent his son Mahinda and daughter
Sanghamitra to Sri Lanka. Ashoka’s Dhamma was not a narrow Dhamma. It cannot be regarded as
sectarian faith. Hence statement 1 is not correct.
• Ashoka held that if the people behaved well, they would attain heaven. He never said that they would
attain nirvana, which was the goal of Buddhist teachings. Hence statement 3 is not correct.
29 www.visionias.in ©Vision IAS

Google it:- https://upscpdf.com


https://t.me/UPSC_PDF Download From > https://upscpdf.com https://t.me/UPSC_PDF

Q 92.A
• The Sufis were organized in 12 orders or Silsilahs. A Silsilah was generally led by a prominent mystic
who lived in a Khanqah (religious home) along with his disciples.
• The Sufi orders are broadly divided into two:
o Ba-shara
✓ Those who followed the Islamic Law and its directives such as namaz and roza.
✓ Chief amongst these were the Chishti, Suhrawardi, Firdawsi, Qadiri and Naqshbandi silsilahs.
o Be-shara
✓ Those who were not bound by Islamic Law.
✓ The Qalandars belonged to this group.
• Hence, option (a) is the correct answer.

Q 93.A
• Sanskrit was the court language of Guptas as a result of which sanskrit literature flourished under
them. Hence, statement 1 is not correct.
• Kalidasa, the great poet, and playwright was in the court of Chandragupta Vikramaditya.
• He composed great epics such as Abhijnanashaakuntalam, Kumarasambhavam, Malavikagnimitram,
Ritusamharam, Meghadootam, Vikramorvashiyam, and Raghuvamsham.
• The celebrated Sanskrit drama Mṛcchakatika was composed during this time. It is attributed to
Shudraka. Hence statement 3 is correct.
• Poet Harisena also adorned the court of Samudragupta. He wrote the Allahabad Prashasti
(inscription).
• Vishnu Sharma of Panchatantra fame lived during this era.
• Amarasimha (grammarian and poet) composed a lexicon of Sanskrit, Amarakosha.
• Vishakhadatta composed Mudrarakshasa.
• Other grammarians who contributed to the Sanskrit language include Vararuchi and Bhartrihari.
• Bhasa wrote 13 plays which echo the lifestyle of the Gupta Era along with its prevalent beliefs and
culture.
• The unique features of the plays of this time is that
o They are all comedies. Hence statement 2 is correct.
o Characters of higher and lower classes do not speak the same language, Women and Sudras
featuring in these plays speak Prakrit.

Q 94.C
• During the Gupta period, the caste system became rigid. The Brahmins occupied the top ladder of
society. They were given enormous gifts by the rulers as well as other wealthy people. The practise of
untouchability had slowly begun during this period.
• The position of women had also become miserable during the Gupta period. They were prohibited
from studying religious texts like the Puranas. The subjection of women to men was thoroughly
regularized. But it was insisted that they should be protected and generously treated by men. The practice
of Swyamvara was given up and the Manusmriti suggested the early marriage for girls.
• There were no restrictions on people’s movements and they enjoyed a large degree of personal
freedom. There was no state interference in the individual’s life.
• The Gupta period witnessed a brilliant activity in the sphere of mathematics, astronomy, astrology
and medicine. Aryabhatta was a great mathematician and astronomer. He wrote the book Aryabhatiya in
499 A.D. Varahamihira composed Pancha Siddhantika, the five astronomical systems. He was also a
great authority on astrology. His work Brihadsamhita is a great work in Sanskrit literature. In the field of
medicine, Vagbhata lived during this period. He was the last of the great medical trio of ancient India.
The other two scholars Charaka and Susruta lived before the Gupta age. Hence option (c) is the correct
answer.
Q 95.A
• Kalinga/Odisha school
o Kaḷinga architectural style is a sub-style of Nagara architecture, flourished in the ancient Kalinga
region - present state of Odisha, West Bengal and northern Andhra Pradesh. Gopurams and Large
water tanks in temple premises are features of Dravidian style of temple architecture. Hence
statements 1 and 3 are not correct.
o In this, Shikhara (called deul in Odisha) is almost vertical before it curves near the top. Shikhara is
preceded by a mandapa (called Jagmohan or ‘dance pavilion’ in Odisha). Hence statement 2 is
correct.
30 www.visionias.in ©Vision IAS

Google it:- https://upscpdf.com


o The style consists of three distinct types of temples: Rekha Deula, Pidha Deula and Khakhara Deula.
The Rekha Deula and Khakhara Deula houses the sanctum sanctorum while the Pidha Deula
constitutes outer dancing and offering halls.
o Examples of Kalinga architecture: Rajarani temple (Bhubaneshwar); Jagannath Temple, Puri.
• Jagannath temple, Puri is known as White pagoda whereas Sun temple, Konark is called Black
Pagoda.

Q 96.C
• Mughal paintings: Mughal School of painting originated in the reign of Akbar in 1560 A.D. Emperor
Akbar was keenly interested in the art of painting and architecture.
• The Mughal style evolved as a result of a happy synthesis of the indigenous Indian style of painting
and the Safavid school of Persian painting. The Mughal style is marked by supple naturalism based on
close observation of nature and fine and delicate drawing. It is of high aesthetic merit. It is primarily
aristocratic and secular. Hence statement 1 is not correct.
• Features
o Mostly or generally miniature paintings
o Use of brilliant colours
o Accuracy in line drawing
o Ornamentation
o Variety of themes use of forsightening technique. Hence statement 3 is correct.
• Under Jahangir, painting acquired greater charm, refinement, and dignity. He had a great fascination for
nature and took delight in the portraiture of birds, animals and flowers.
• Aurangzeb was a puritan and therefore did not encourage art. Painting declined during his period and
lost much of its earlier quality. Hence statement 2 is not correct.

Q 97.D
• Nagara Style of temple architecture is usually related to north India. Some features of Nagara style of
temple architecture include-
o Latina/rekhaprasada: There are many subdivisions of Nagara temples depending on the shape of the
shikhara. There are different names for the various parts of the temple in different parts of India;
however, the most common name for the simple shikhara which is square at the base and whose walls
curve or slope inward to a point on top is called the latina' or the rekha-prasada type of shikhara.
o Phamsana: Phamsana buildings tend to be broader and shorter than latina ones. Their roofs are
composed of several slabs that gently rise to a single point over the center of the building, unlike the
latina ones which look like sharply rising tall towers. Phamsana roofs do not curve inward, instead,
they slope upwards on a straight incline. In many North Indian temples, the phamsana design is used
for the mandapas while the main garbhagriha is housed in a latina building.
o Valabhi: These are rectangular buildings with a roof that rises into a vaulted chamber. The edge of
this vaulted chamber is rounded, like the bamboo or wooden wagons that would have been drawn by
bullocks in ancient times. They are usually called ‘wagonvaulted buildings’.

Q 98.A
• The RBI set up a five-member committee under K.V. Kamath in August 2020 to recommend eligibility
parameters for restructuring stressed loans.
• The RBI had formed a five-member committee under the chairmanship of Kamath to make
recommendations on the financial parameters to be considered for the one-time restructuring of
loans impacted by the COVID 19 pandemic. Other members of the committee are former State Bank of
India executive Diwakar Gupta, current Canara Bank chairman TN Manoharan, consultant Ashvin Parekh
and Indian Banks' Association (IBA) CEO Sunil Mehta who was also a secretary to the committee. Hence
option (a) is the correct answer.
• The Kamath panel has recommended financial ratios for 26 sectors which could be factored by lending
institutions while finalizing a resolution plan for a borrower. The financial aspects include those related to
leverage, liquidity, debt serviceability.

Q 99.A
• The Indian Navy is set to acquire 24 new submarines under the Mega Project which has been
named P-75 I. This is being done to bolster India’s underwater fighting capabilities. The submarines are
to be built in India. Of these six are nuclear attack submarines and are to be built at a cost of Rs 55,000
crores. T Hence option (a) is the correct answer.
31 www.visionias.in ©Vision IAS
• Presently the Indian Navy is operating two different types of submarines — Russian Kilo-class and
German Type 209 conventional submarines. And one ‘Scorpene’ class submarine is the new one
inducted in the Indian Navy. Recently, India banned the import of 101 military items such as cruise
missile, sonar systems, combat helicopters, conventional submarines in order to boost indigenous
production of defence items in the country.

Q 100.D
• Both statements are not correct: India remained the world’s second largest arms importer during
the period 2015-19, with Russia being the largest supplier though Moscow’s share of the Indian
weapons market declined from 72% to 56%. India was the world’s second-largest arms importer from
2014-18, ceding the long-held tag as largest importer to Saudi Arabia, which accounted for 12% of the
total imports during the period.
• However, Indian imports decreased by 24% between 2009-13 and 2014-18, partly due to delays in
deliveries of arms produced under licence from foreign suppliers, such as combat aircraft ordered from
Russia in 2001 and submarines ordered from France in 2008
• Russia accounted for 58% of Indian arms imports in 2014–18, compared with 76% in 2009-13.
Israel, the U.S. and France all increased their arms exports to India in 2014-18. However, the Russian
share in Indian imports is likely to go up sharply during the next five-year period as India signed several
big-ticket deals recently, and more are in the pipeline. These include S-400 air defence systems, four
stealth frigates, AK-203 assault rifles, a second nuclear attack submarine on lease, and deals for Kamov-
226T utility helicopters, Mi-17 helicopters and short-range air defence systems.

32 www.visionias.in ©Vision IAS

You might also like